Anda di halaman 1dari 161

1

9th,10th & 11th May - FCPS-1 -2016 Papers (1650 MCQS)


[ Index- Check List ] Compiled by : Amlodipine Besylate
(1) Medicine & Allied 9th May 2016 (Morning Session) by Haris Iqbal (176 MCQS) Page#1
(2) Medice & Allied 10th My 2016 (Morning Session) by Aish Choudri (181 MCQS) P#19
(3) Medice & Allied 10 My 2016 Evening by Sarah Khan (176) page#29
(4) Medicine 11th May(Morning) by by Usman Dar (192 MCQS) P#36
(5)Surgery & Allied 9th May (Evening Session) by Syed Arsalan Akbar (186 MCQS) P#54
(6).Surgery & Allied 10th May (Morning Session) by Zia Ur Rehman (71 MCQS) P#86
(7). Gyane/Obs 10th May- 2016 (Evening Session) by Dr-Haya Sayed (150) P#92
(8). Gyane/Obs 11th May-2016 (Evening Session) by Nourin Hameed (180) P#100
(9) Anesthesia 9th May Morning by by Saqiba Tahir (176) Page#120
(10) Radiology 10th May (Morning Session) by Bint-e-Adam (162MCQS) P#128
=-=-=-=-=-=-=-=-=-=-=-=-=-=-=-=-=-=-=-=-=-=-=-=-=-=-=-=-=-=-=-=-=-=-=-=-=-=-=-=-=-=-=-=-=-=

(1) Medicine 9th May 2016 (Morning) by Haris Iqbal (176)


I tried my best to recall nd error free. I compiled these very hardly bcz i m not good at it Right Ans have star mark...!!!!

1..pt with toothache on warfarin planned for surgry,wht test will u do


A...pt,aptt,IN*
B..pt,aptt
2..how cimetidine increases warfarin effect
A...dec hepatic clearance*
B...removing from plasma
3...pt on warfarin,how ll u monitor
A...pt*
B..aptt
4...ulnar nerve damage at elbow
A...claw hand deformity*
B...hypothenar wasting
5...Aspirin MOA
A...inhibit prostaglandin production*
B..inhibit phosphoralase
6...knee relex root
A...L3L4*
B..L4L5

7...Asiatic cholangioheatitis
A...c sinenses*
B..giardia
8...glycogen store more in which organ
A...liver*
B..skeletal muscle
C...smoth muscle
9...Adenoma
A...glandular type apperance*
B...arises from serosa
10...anion gap
A...difference b/w unmeasured anion and measured cation*
B...diff b/w measured anion nd unmeasured cation
11...vit D deficiency
A...ricket*
B...paget D
12...inferior thyroid vein drain into
A...left brachiocephalic*
B..int jug vein
13...scalp 4th layer
A...loose areolar tissue*
B..priostum
C..muscular tissue
14...ankylosing spondylitis
A...HLAB27*
B...HLAB4
15...Definitive diagnosis of tb
A...afb*
B..caseous granuloma
C...epitheliod cell
16...microscopic finding of tb
A...caseous granuloma*
B...epitheloid cells
17...heart act as a synctium due to
A...gap junction*
B...tight junction

18...S2 different from S1


A...frequency*
B..duration
19...S1 produced by
A...isovolumetric contraction*
B...isovolumetric relaxation
20...retes ridges with other features
A...squamous cell ca*
B...verrocous ca
21..cimetidine frequent side effect
A..hepatic enzyme inhibition*
B..nausea vomitting
22..ear infection by pseudomonas,fever caused by
A...TNF*
B...iL1
C...iL5
23...neural tube defect
A...meningocele*
B...herniated disc
24...klinfelter syndrom
A...47xxy*
B..46xxy
25...scenerio in which total bilirubin increas nd direct normal.cause
A...gilbert syndrom*
B..ovstructive hepatitis
26...after stroke,how will ischemic brain tissue repaired by
A...Neuron
B.. fibroblast
C...schwan cells
27...which neurotransmitter increas will dec anxiety
A...gaba*
B...glutamate
28...deep sleep characterised by
A...bruxism
B...active dreaming
C...penile erection

29...Area 1 different from motor area


A...granular layer*
B...homunculus
30...basilary artry branch
A...pca*
B...mca
C...aica
31...Lp pieces which structures
A...dura,subdural space,subarachnoid membran,pia*
B...subdual space,dura,pia,subarachnoid
33...spinal cord supplied by
A...vertebral artry*
B...intercostal artries
34...phrenic nerve relation
A...pericardiophrenic Artry*
B..esophagus
C...great cardiac vein
35...primordial germ cell
A...6 week*
B...3week
C...2week
36...typhoid ist week test
A...blood culture*
B...widal
37...gentamicine
A...contraindicated in renal impairment*
B...doc fr uti
C...increases hepatic clearance
38...contact dermititis
A...delayed hypersensitivity reaction*
B...type3 HR
C...type2 HR
39...female blood group A,two child with blood group O,AB.wht will be father blood group.
A...B*
B..Ab
C..O

40...pulmonary thrombo embolism most commonly from


A...popliteal vein*
B...great saphenous vein
C...pelvic veins
41...female while climbing stairs have sudden chest pain nd dyspnea
A...V/Q mismatch*
B...MI
C...COpd
42...intraceolomic derived from
A...extraceolomic mesoderm
B...paraxial mesoderm
C...lateral plate mesoderm
D...from ntochord
43...cuboidal epithelium
A...salivary gland ducts*
B..male urethra
C..collecting ducts
44...bulbar urethral rupture blood extravasat
A...superficial perineal pouch*
B...deep p pouch
C...scrotum
45...corneal reflex by
A...opthalmic branch of trigeminal N*
B...occulomotor N
C.. optic N
46...pt having copd with respiratory alkalosis,renal excretion of potassium will be
A...fal k,due to increas tubular fluid
B...rise k,due to k nd acid coupled*
47...female fall nd presented n ER with short left leg nd lateral
A...hip dislocation
B...inferior gluteal N damag
C.. sup gluteal N damag
48...aschoff bodies first feature
A...fibrinoid necrosis*
B...eosinophil
C...giant cell

49...small cell lung ca on histopathology which cell


A...acth*
B...pth
50...in lactation GnRH inhibition by
A...prolactin*
B...GH
C.. oxytocin
51...lipogenesis,glycolysis is a function of which hormone
A...insuline*
B..Gh
C..cortisol
52..hyperfunction thyroid effect on heart
A...tachycardia*
B...increas pR interval
C...vulvular disease
53...after partial gastrectomy which will happen
A...hypochromic megaloblastic anemia*
B..iron deficiency anemia
C..normochromic megaloblastic anemia
54...an anemic pt have microcytic anemia picture nd his elder brother have transfusion hidtory
for same problem.which Ix will u do
A...hb electrophoresis*
B...serum ferritin
C...serum vit b12
55...all antisychotic same in
A...antiemetic effect*
B...efficacy
C...potency
56...asthenuria due to
A...SIADH*
B...ADH
C...ALDOSTERON
57...Neyrohepatic malignant syndrome by
A...Haloperidol*
B..Reserpine
C..cimetidine

58...Drug excretion depend upon


A...glomerular filteration*
B...fat soluble drugs increase
59...adult Ards was treated with porcilin(surfactant),wht was its MOA
A...dec alveolar surface tension*
B...increase complaince
C...increase intrapleural pressure
60...Edema of renal origion
A...albuminuria and salt retention*
B...salt retention
61...Neuron RMP decreases by
A...k*
B...Na
C...cl
D...ca
62...which one is passively absorbed from renal tubules
A...cl*
B...NH3
C...glucose
63...obstructive disease of lung increases
A...FRC*
B...FEV1
C...VC
64...absent A wave due to
A...mitral stenosis*
B...aortic stenosis
65...Dubai taxi driver with fever,malaise nd lymphedenopathy nd other immunosupressive
features
A...HIV*
B...yellow fever
C...malaria
66...Eosinophila
A...hogdkin Disease*
B...non hodgkin D
C...tb
67...LH act on which testicular cells mostly
A...leydig cells*

B..sertoli cells
C...spermatogonia
68...Neural crest derivative
A...Sensory ganglion*
B..astrocytes
C..oligodendrocytes
69...female pt pap smear shows squamous cells in endocervix
A...metaplasia*
B...dysplasia
C...hyperyrophy
70...2 feet away on antimesenteric border there is a dilated structure
A...mickel,s diverticulum*
B...hushprung disease
C...adenoma
71...right bronchial artry is branch of
A...posterior 3rd intercostal artry*
B...descending aorta
C..aorta
72...vagal stimulation causes
A...increase PR*
B...increase RR
C...Decrease PR
73...child have non fever diarrhea for maney days,on microscopy curved nd gram negative
bacteria
A...vibrio*
B...E coli
C...Giardia
74...ameboid locomotion
A...due to cilia
B...helps in germ cells migration*

75...respiratory symptoms by
A...ascaris*
B...t solium
C...A deudenale

76...Nitroglycerine MOA on vessels


A...cgmp stimulation*
C...inhibtion of beta receptors
77...osmolarity controlled by
A...feedback of osmoreceptor*
B...ADH
78...adrenal insufficiency main feature
A...Hyperkalemia*
B...hypokalemia
C...hypernatremia
79...Terminal bronchiol
A...pseudostratified epithelium*
B..more goblet cells
C..mucus secreting cells
80...Right kidney anterior relation
A...right hepatic flexur*
B...liver
C...deoudenum
81...which structure arch over aorta
A..left RLN*
B...left bronchus
C...left phrenic N
82...seconadary peristalsis due to
A...esophagus dilatation*
B...only due to achalasia
C...due to metaplasia
83...CCk and gastrin are similar due to
A...same terminal C group*
B...same function on stomach
84...fetal hemoglobin
A...increase affinity for O2*
B...only found in fetal life
C...
85...iron absorption from gut increases by
A...iron ferrous form*
B...hCO3
C...gastric acid

10

86...soldier on hilly area for 6 months came back with the conplaint of cyanosis
Nd other features
A...secondary polycythemia*
B...polycythemia vera
C...methemoglobinemia
87...worker of mine nd sand blaster have dyspnea
A..silicosis*
B..anthracosis
C..asbestosis
88...which is more related to malignancy
A...asbestos*
B...berrylium
C...aromatic amines
89...left heart failure due to
A...aortic stenosis*
B..pulmonary HTN
C..corpulmonale
90...thermorecetors
A...long receptive field*
B...special receptors
91...S1 due to
A...isovolumetric contraction*
B...isovolumetric relaxation
92...calcium channel blocker which act on SA node
A...verapemil*
B...diltiazem
C...nefedil
93...SA node supplied by
A...RCA*
B...marginal branch
94...epinephrine do vasodilation by acting on which receptor
A...beta 1
B...bet 2*
C...alpha 1
D...alpha 2

11

95...postmenupausal women have


A...increase LH nd FSH*
B...Dec LH nd FSH
96...seductive female pt came near to you,wht will u do
A...call in nurse*
B...dont examine her
97...medical ethics
A...code of conduct of dr profisional life*
B...doctors right
98...three percentages were given
Which chart
A...pie chart*
B...bar chart
99...which antiemtic will u prefer in chemotherapic pt
A...ondenstran*
B...metoclopromide
100...HIV related skin cancer
A...kaposi*
B...sq cell ca
101...Exudate contain
A...inflamatory cells*
B..increase lipid
102...Decrease ESR
A...increase albumin*
B..Dec albumin
103...vasculitis related with hepatitis
A...hep B*
B...hep C
C...hep E
104...chronic hepatitis is characterised for how maney duration
A...6 month*
B...3 month
C...2 month
105...tumors T4N1M1 having common features
A...cachexia*

12

B...wieght loss
C...anemia
106...thymus
A...Rich in lymphocytes*
B...regress immediatly after birth
107...cardiac muscles
A...autonomic supply*
B...striated
108...Initial investivation of leprosy
A...nasal scraping*
B...culture
C...biopsy
109...icu infections prevented by
A...multiple hand washing*
B...using face mask

110...scenerio of a child with petechae nd bruises with bt prolong


A...iTP*
B...hemophila
C...platelet functional defect
111...potent antioxident
A...vit E *
B...vit A
C...vit C
112...fibrillin mutation causes
A...marfan syndrome*
B...sideroblastic anemia
C...duchne muscular dystrophy
113...Lp done on a pt,now he conplains of headache
A...dura mater comression*
B...falx cerebri compression
114... reversible cell injury
A...membrane blebs
B...swelling of E reticulum*

13

115...Neostigmine MOA
A....inhibitor of cholinestrase*
B...anticholinestrase
116...Mitral stenotic ECG changes
A...p mitrale
B...irregular RR
117...secondary cartiligious joint
A...symphysis pubis*
B...chostochondral jiont
118...pt of epidermis burn is at risk of
A...contracture*
B...granulation
C...malignancy
119...hyaline cartilage in
A...larynx*
B...articular surface of tmj
120...Icam nd vcam are
A...adhesion molecules*
B...rbc proteins
121...wBc adherence to damage endothelial wall
A...margination*
B...migration
122...factor 8 produced by
A...endothelial cells*
B...hepatic cells
123...damage r abnormal protein is degraded by
A...lysosomes
B...golgi apparatus
C...peroxisomes
D...polyphagosomes
124...appropriate about thrombus formation
A...damage valve,dec blood flow,increase viscosity
B...damage valve,dec blood flow,wbc*
C...damag valve,wbc,increase flow

14

125...glucocorticoides causes
A...dec peripheral utilisation of glucose*
B...increase glucose
126...conn syndrome
127...after MI in minutes which occure
A...Arrythmia*
B...cardiac temponade
128...resonance on back upto
A...t8*
B...t10
C..t6
129...there is small cystic mass in left parietal lobe with involment of heart features
A...myxoma*
B...glioma
C...cardiac hypertrophy
130...which is anticancer
A...apple*
B...chocolate
C...nitrated meat
131...when membrane potential decreases
A...k goes out*
B...inevitable action
C...cl goes in
D...k goes in
132...Neurotransmitter released in chemical junction by
A...ca influx*
B...ca efflux
C...na influx
D...hyperpolarisation
133...Tissue blood increase due to
A...decrease bp
B...increase pO2
C...increase pCO2
134...heavy bleeding in treacheostomy due to
A...a vein which drain into jugular vein*
B...inferior thyroid artry
C..due to isthmus

15

135...thyroid isthmus lie at


A...Treacheal ring 234*
B...T Ring 345
136...gut affected from hepatic flexure to mid transvers colon,artry affected
A...midle colic artry*
B...iliocolic artry
C..sma
137...anterior surface of heart formed by which structure
A...rt ventricle*
B...rt atrium
C..lft ventricle
138...spinal cord supported by
A...denticulate ligamet*
B...transverse ligament
C..ligamentum flavum
139...sense of smell
A...afferent nerves in nasal cavity
B...auditory cortex
C...passes through thalamus
140...female pt with b/l pleural effusion nd dyspnea and having pedal edema
A...recurrent thrombo embolism*
B...dvt
C...chf
141...axillary sheath formed by
A...prevertabral fascia*
B...pretreacheal fascia
C...superficial layer of cervical fascia
142...p falciparum feature
A...black water fever*
B...jaundice
C...fever
143...increase in 2/3 of thoracix diameter by
A...bucket handle effect of ribs*
B...diaghragm
C...handle effect of ribs
144...chronic hep C pt on microscopy cells with eosinophilic cytoplasm nd other features
A...apoptosis*

16

B...fibrosis
C..necrosis
145...cyanosis due to
A...deoxyhemoglobin 5g*
B...increase pCO2
C...increase pO2
146...child with yellowish sclera
A...alt plus bilirubin*
B...alt
C...alt nd ast
147...diabetic nephropathy Ix
A...urine albumin*
B...urine glucose
C...urine create
148...during peristalsis when lower esophageal sphincter relaxes
A...before bolus reach
B...after bolus reach
149...which thing in salivary mucus incorporate micro organisms
A...lactoferrin*
B...lysozyme
C...lipase
150...which is true both about immunity and allergy A...Aids decrease T cells

151...GVA
A...viscera*
B...glands
C...other organs
152...parotid gland supply
A..GVE*
B...GvA
153...louder S1
A...mitral insufficiency*
B...lower limit of PR interval
154...gastric ulcer pain
A...greater splanchnic N*
B...lesser splanchnic N

17

155...great vein of galen located in


A...superior
B...inferior
C...middle
156...blood cells of bone marrow get signals to proliferat from
A...BM endothelial cells*
B...Bm fat cells
C...erthropoitin
C...growth factor
157...SLE caused due to
A...B cell dysfunction
B...T nd B cells dysfunction
C...immunological response to hydralazine
158...inspiratory Neuron
A...apneustic*
B....pneumotaxic
159...Nitroglycerin not goven orally bcz
A...extensive ist pass effect*
B...bioavailibility is low
C...cant absorbed from gut
160...in Acute pancreatitis peritonium will show
A...fat necrosis*
B...wet gangrene
C...dry gangrene
161...most potent stimulus for erythropoitin
A...hypoxia*
B...aldosteron
C...dec flow
162...11 year girl x ray elbow age will show A...lateral epicondyl* B...med apeicondyle
163...Major buffer of blood
A...HCO3*
B...hb
C...h2co3
164...2*2 A...chi square test* B...t test
165...after adrenelectomy increase taste for
A...Nacl*

18

B...cacl2
C...nahco3
166...for transplant HLA,Best cells are
A...wbc*
B...rbc
C...kidney cells
D...hepatic cells
167...G6Pd disorder most commonly
A...drugs induced*
B...genetic
C...immunogenic
168...Mosquito bite
A...yellow fever*
B...filiriasis
C...leishmaniaisis
169...post syphilitic pt cant constrict pupil on light,with intact accomodation
A...argyl robertson pupil*
B...masthenia gravis
C...occulomoter palsy
170...lung base have high perfusion nd ventilation why
A...high blood flow*
B...low blood flow
C...high diffusing capacity
171...force of contraction of heart determined by
172...chronic alcoholic with normal LFTs on microscopy liver will show
A...fatty changes*
B...normal
C...fibrosis
173...one about Neuron RMp
174...One about motor area
175...2 about ecg RR nd PR intervals
176...one about sarcoidosis,asking about type of cell involved
Ps...Others cnt remember nd recall Regards...Dr Haris Iqbal .....keeep remember n duwazzz....
Best of luck....

19

=-=-=-=-=-

(2) Medicine 10th May 2016 (Morning) by Aish Choudri (181)


1.a child with ocular anomlies,corneal defects,lense prolapsed.dia..trisomy13
2.condition vd normal po2 but dec o2 saturation.CO poisoning
3.hypoglycemia lead to inc level of ..Glucagon
4.loss of thenar eminence,opposition of thumb ...median Nerve
5.sup cardiac plexus by left vagus
6.bitemporal hemianopia lesion chiasm
7.charctristic of TB ...epitheloid cells
8.ACTH secreted via .small cell CA
9.PERSON standing for a long time .cvp decreased
10.REcurrant lyrngeal N around

ligamentum artriosum

11.dec visual acuity in thyroid disease due to .. optic Nerve compression


12.sick role

..

..,. abstinence excuse from resposibility

13.diff in plasma nd Interstitial fluid osmolairty 1mosmo


14.BODY response in cold to regulate temp

..hunger

15.highest con of acth in stress seen in .. ant pituitary


16.retarded bone growth fracture via .epiphyseal plate
17.SCM supply spinal .accessory Nerve

20

18.muscle of fright platysma


19.proband .first person who seek medical advise
21.structrelateral to ICA jugular vein
22.Person cant move his foot medially (tibialis ant +pos)
23.contact dermatitis by latex gloves. type 4 hypersensitivity
24. Reaction to immunoglobulin in ITP patient after 1 week..type 3 hypersensitivty
25. in myasthenia. igG
26 Nerve damged thats content of cavernous sinus ..abducent
27.vWF.platlet adhesion
28.omeprazol causesH/K pump inhibition
29.complete heart block.. fainting due failure of ventricles to contract
30.psychophysical effct of brain activity ..papillary reflex ( confirmed)
nbihttps://www.google.com.pk/url?sa=t&source=web&rct=j&url=http://www.ncbi.nlm.nih.gov/pmc/
articles/PMC3831461/&ved=0ahUKEwiW_q_TitXMAhXLLY8KHTLhBt4QFggnMAQ&usg=AFQjCNGP
51vWGXtt026gvq5efLLe-ANfHw
31.DKA mucormycosis..
32)normal distribution curve ...gaussian curve
33. renal>>interlobar>>arcuate>>interlobular>>afferent>>glomerulii
34.adh and vaso originate in hypothalamus.
35)hyperoxaluriadue to dec calcium level to form complexes vd oxalates
36)b/l hippocampus damage .loss funtion to convert recent memory to long term

21

37)most common primary immunodef is..common variable immun deficiency also called B cell
deficiency
38.child with repeated infection due to def of .igA
39.NEUTOPHILL ..phagocytosis
40.oxidative burst opsonization
41.muscle in quite respirationdiaphragm.
42)max blood loss during hypovolmic shock thru .. veins
43)1/3 is . ECF
44)afp raised in. embroynal CA.
45)HEART RATE is 40bpm complte heart block
46)inc output failure due to.. thiamine def..
47)cholngioCA organism ,.clonchris sineseis
48)4year boy splenctomy done. strep pnemoccoal vaccination
49.bee sting but no bronhospasm chest clear I M adrenaline
50)cyanide block ..cytochromeP450 enzyme
51)6 week baby diaper stained. billary atresia
52)left shifting curve ..increase Ph
53)60yr lady bluish spot on thigh,BT 1 MIN,CT 3min ..platlet defect
54.Right side touch lft side temp pain loss.lesion right hemisection
55)right hemicolctomy marker tumor marker CEA..

22

56)THROID CA histo related long scenario at end asking marker .calcitonin


57)basophiillia to cell on histo..RER
58)anticoagulant contraind in.. thrmbocytopnia
59)esophageal natural constriction vhere crossed by aortic arch
60)protamine side effct hypotension
61)TCA vmain side effect . tonic clonic seizures
62)commonest factor of cirrhosis in young adults overall .alcohol
63)aspirin overdose ..alkalosis
64)pt of asthma nd HTN fr ER give.. i/v nitropruside
65)asthma vd emphysema vat wil .dec fev1/fvc
66)most common virus caused hepatitis in our country. HBv as hav was not in option
67)IN obstructive lung disease what vill b inc ..inc elastic recoil
68)thyroid ..pretracheal fascia
69)facial palsy and intention tremers prob cerebellopontine angle
70)inc t4 nd pt is intolerant to heat what will dec TSH
71)Megaloblastic anemia diphylobothrium latum
72)anemia pins,needles sensation dec platlets decreased wbc aplastic anemia
73)broca supplied by MCA
74)pos 1 /3 tongue sensations lost damage of.. glossophyrngeal nucleus
75)carcinoid syndrom .indol ascetic acid

23

76)serotonin syndrome caused by ..TCA vd fluxteine


77)40yr non diabetic female vd ogtt in morning 122,aftr 1hr 198,aftr 2hr 192 etc Impaired glucose
tolerance
78)iron transport thru ..transferrin
79) atlantoaxial ..pivot joint
80)regarding sympthtic activation what will epinprine do inc skeletal muscle flow
81)FEMALE vd irregualar periods fsh;lh 3;1 .PCOD
82)TUmour that dont secrete any hormone .teratoma
83)lordosis .. ant convxity
84)anaphylactic shock diff from hypovolemic inc CO
85)niesseria . benzyl penicillin
86.infction on gram stain gram pos in clusters ..pencillin
87)drug requiring serum monitoring. vancomycin
88)poor prognosis of sepsis is DIC
89.long scenario,lady from remote village typical oneDIC
90)simple diffusion not saturable
91)liver transplant,size inc due to.. hyperplasia
92)hypertrophy nd hyperplasia together .uterus in pregnancy
93) abduction lost upto 40. lost supraspinatus
94)inc in right atrial pressure in normal limits ..increase cardiac ouput

24

95)angina like symptoms in aortic stenosis


96)seondry center of ossification in epiphysis
97)malignancy indicator is metastasis
98)tumor metastasis due to .degnration of E cathedrins
99)pt vd difficulty walking raising leg left pelvis sink down....SUPERIOR GLUTEAL
100)MALE vd regular checkup b.p 170/110 didnt took treatment aftr one mnt 165/110 in long run
inc risk of ..hyperplastic arteriosclerosis
101)in which phase chromosomes are arranged in chromatid .metaphase
102)man in shop suddenly collapsed 120bpm bp 90/60,frst had excursiating chest pain
PULOMNARY EMBOLISM
Sudden collapse always think pulmonary embolism
103)angiotension cause

vasoconstriction

104)athltete has inc which one in normal state. inc cardiac output
105)neurovascular bundle. lower border of rib
106)wernickes area nonsense fluent
107)csf question option were difficult and different
108.most common place of obstruction in hydocphlus ..aqueduct of sylvius
109)crf,hyperphosphatemia
110)fetal period aftr .8 weeks
111)veins thrombophelbitis

25

112)Common hepatic duct join cystic duct


113)space between splnchic and somatic mesoderm is extraembryonic
114)thromboembolism causative factor endothelial injury
115) immunoglobin produced by plasma cells
116)LVF by .aortic valve lesion
117)l dopa effct start decreasing due to degradation of receptors substantia nigra
118)PCT distal part .. urea conc is greater in distal PCT than glomerli
119)heparin inhibt .clot propagation
120)DM ..angiopathy and neuropathy
121)appendix position.. retrocecal
122)in TB ..inc lymphs
123)mononucleaosis ..monospot test
124)baby with A POSITIVE blood grouperythroblastosis fetalis blood transfusion A ngtve
125)sclerderma pt vd enuresis drug . oxybutonin .
126)left ventricle relations base is. pos perforated substance
127)thirst inc by.. dec ecf
128) glucose absorbed .PCT
129)power stroke during skeletal muscle contraction ..Myosin
https://www.boundless.com/biology/textbooks/boundless-biology-textbook/the-musculoskeletal-system38/muscle-contraction-and-locomotion-218/atp-and-muscle-contraction-826-12069/

26

130)only pt allowed in ER
131)injry spontaneous penumothorax .pleural pressure equal to atmospheric
132) pain in joint in ATT worsen condition..pyrazinamid
133)RECURRANT abortions antiphopholipid antigen
134)hyperkalemia related question of dec potassium secretion .addison disease
135)diarrhea metabolic acidosis .normal ion gap
136)parietal cells
137)high hcl require for pepsinogen to pepsin
138)recurrant lyrngeal ..branch of vagus
139)atropine block sweat glands
140) emax related to. Efficacay of drug
141)true hermaphrodite

.xxy

142)diagnostic for SLE anti ds DNA


143)ANA .. vasculitis
144)growth hormone stimulated by hypoglycemia
145)steroids dec .lymphocytes
146)whch cell inc in viral infction... lymphocytes
147)babnski positive lost voluntary activity on rt lesion at area4
148)high carbohydrate in.. wheat
149)giant cell >>fibriniod necrosis

27

150)tumor dec what to prevent killing mhc1


151)allels..co dominannce
152) frst effected if bronchigenic CA effect bronchus. Superior apical
153.bronchiognic CA smoking
154.vibrio opening.. cl channels
155 premalignnt. Bowen disease
156.quack gave some medicine monitor.AST Alkohol/quack medicatio=AST Viral / toxin =ALT
157)BLOOD return inc via contraction of calf muscles
158.lymph node biopsy of HIV finding .. T cell depletion
159)menupause dec estrogen inc fsh nd lh
160)plasma membrane thikness. 8nm
161.girl doing exercise most fluid loss via sweat
162.kawashirkor scenario
163.paking and transport of carbohydrate protein. golgi complex
164)good factor fr determing disease progressionsenitivty
165)epicardium connective tissue
166)fracture of pelvis shock due to .blood loss from veins
167)feature of first cervical vertebrae no body
168)bladder regarding ..s2,3,4
169)micturation ..detruser muscles

28

170)pos belly of digastrics. facial nerve


171 cholesterol lipoprotein..LDL
172)shock adrenaline
173.thigh swelling. veminton
174.diazepam nd chlorpromazine is. sedation
176.one about git and pancreas etc
177.melanoma on thigh

medial group of suoerficial inginail nodes

178.rashes in descending order measles


179.ischemia lead to ..coagulative necrosis
180. v warm sensations activate ....... pain fibers
181.inflamtion mediator. bradykinin
Feel free to correct it if u r sure about any questions wrong in it.

I wrote options against all ambiguous questions,recall error may b there

Thanks to mian behram,Muhammad athar malik,hussam ud din,ghazi muhammd,mehreen Fatima and everybody else who
participated in it Remember in duas specially.

=-=-=-=-=-=-=-=-=-=-=-=-=-=-

29

(3) Medicine 10th May 2016 (Evening) by Sarah Khan(176)


1 .Dural sensation lost in middle cranial fossa nerve involved

trigeminal nerve

2.Immunoglobin in erythroblastosis fetalislgg


3 in case of rubella antibodies in pregnant female indication for pregnancy terminationigm
4. parent reactiondenial
5. first thing rise after splenectomy thrombocytosis
6. scanty barr bodiesturner
7. barr bodies test is diagnostic for Kleinefelter
8. b cell marker CD19 & CD 20
9. about hyaline cartilage COLLAGEN NOT VISIBLE
10. cavernous sinus contain internal carotid and abducant
11. antipsychotic similar in property ..efficacy
12 Azathiaprine xanthine xidase
12. atypicall cell in IM T cell
13. cell infected in IM

B cell

14. diagnosis of IM testmonospot


15. female with hoarseness mild dysphagia neck mass tumor markercalcitonin
16. testicular yolk cell tumortumor markerAFB
17. trochlear nerve damage DOWNWARD GAZE ADDUCTION
18. ovarin pain radiating to medial thigh nerve involvedobturator
19. RTA patient vitally unstable initial stepfluid
20..mhc regarding question dont remember
21..hla related .associated with graft rejection
22. folic acid present inraw vegetable
23. permanent Dilated pupil nerve,,,occulomotor
24,related to GTO
25.obs jaundice in pregnancy GGT

30

26. hep b monitoring SGPT


27. lesion of optic chiasmabitempral hemianopia
28. optictarac right sided lesionleft homnymus hemianopia
29. indolacetic acid in carcinoid
30.secretin selectively inc h2co3 from pancreas
31. PPIinc gastrin sec
32.estrogencontaing pills thromboembolism
33. patient having dysnea no cough rvh prominent vascular marking what seen in pulmonary artery
atherosclerosis
34. throbus like lesion in left atriummyxoma
35. esophageal compression due to left atrium enlargement
36.all values were normal like calcium phosphate only alp was raised
37helper cell function help in antibody formation
38..bp high sugar present in urine inblood normal pottasiun low cause of htn..primary
hyperaldosteronism
39. maculadenasense low sodium what will release..renin
40.lumbar puncture above L4
41. prolactin raised with infertility wat will be cause microprolictinma Macroprolactinoma
42. ocd doc clomipramine
43.valvues given with combination correct to choose bout csf
Bbacterial low glucose ans
44. plasma and csf comparisonlow glu in csf
45..plateletagession due to rough endothelium
46..ser apart from steroid synthesis do whatdetoxification
47.plasma cell disorderamyloidosis what will be present..amyloid light chain
48.5 year old with precious puberty

cah

49.type 3 hypersentivityafter some diphtheria toxoid 3 weeks later


50. not included in intrincic pathway yfactor7
51. ppd which cell present cd4 and macrophages

31

52. I vivo factor that is activated and activate other factores,,,THROMBIN


53. myocarditisscanerio organism involved cocksackie virus
54. insulin sec inc by GIP
55. pancreatitis plus neuropathy drugdiadinosine
56,,regarding pancreas tail in splenorenal ligament
57. fat necrosisin peritoneum
58. farmer unconscious low sodium causeexcessive sweating
59..brain liquefactive necrosis
60. fasting man concentrated urine cause.adh
61.Post communicating artery pass above and connect with pca
62.plasma low osm urine high osmcause SIADH
63. lateral tract sensation lost on left while dorsal sensation on right..cause
64.aldosterone act on cortical collecting duct
65. what will be present in anaphylactic shock as compared to hypovolemic shocinc cardiac out put
66. uml of facial nerve,,,spare upper half of face
67. bellpasly lesion STYLOMASTOID ANGLE??
68. prolong commaperieqiduact of gray+?
69. dorsal columnpropioception
70. taste sensation final relay???
72.decending path stimulate alpha and gamma simultaneously VOLUNTRY CONTRACTION
73.RELATING TO MUSCLE SPINDLE when muscle contract
74. loss of abduction adduction of fingers and loss of abduction of thumb and intact sensationnerve
involveddeep ulnar nerve
75..rt horizontal call stimulationrt side movement of head
76. gardner with lung lesion ..aspergillosis
77. wegener,s scanerio..nose lesion and oroteinurea
78.diabetic nephropathy inv..urine alb
79.child with edema ..first inv urine protein

32

80. local edema .allergy


81.chemical carcinogenbenzidine
82.2nd day bay with jaundicephysiological
83. muscle cause foot eversiobnp longus
84. inter cosatal neurovascular bundle arrangement from cranial to cadualVAN
85. arrangement inn femoral trigangle from medial to lateraVAN
86. regarding fontanelleant bigger than post
87, thyroid enclosed inpretracheal facia
88. inf from pretracheal spread to,ant mediastinum
89. arterysuppluubroca areaMCA
90..max salt and water absjejunumcolon
91.crons different from ulcerative periabnall lesion
92. 100 cm ileum ileumrection..dec bile salt absorption
93. heart sount..s1 isovolumetric contraction
94. non fitting dentures dentist says no dental problem..lesion in sub nigra
95. at puberty what happen in gnrh
96.dead spacehumidify and filter inspired air
98. ductusarterious connectarch of aorta left pulmonary artery
99 100 percent po2 in left to right shunt
100. in co and methemoglobinemia why hypercapeadoent occurpo2 normal
101. cervical rib compressT1
102. radiation injury depend upon..duration of radition
103. about blood supply of heart
104. phrenicnereve accompany ..pericardiophrenic artery
105. on right side of trachea what is present ,,,vagus nerve
106. teacher has tremor with family hx all labs normaldoc propranolol
107. pregnancy recurrent abortionantiphospholipid
108. anticoagulant in pregnancy ..iv heparin

33

109..parasympathetic drug effectdec hr by inc K conductance


110. alpha adrenic stimulation.pupil dilation
111. fev1 ratio fvc=0.8
112. dull percussion aspiration sjhow s inc neutron ..purulent exudate
113..crudy discharge pseudo hyphaecandida
114.hlab27 ankylosing
115..sjogrensyd.. RA
116. pregnant lady htyn before surgery what to givehydralazine
117. incidence def
118 prevellance def
119.inc esophageal tone and motilitymetoclopramide
120..ovarian cancer tumor marker ca 125
121,, chronic hepatitis fibrosis or lymphocyte
122..sma branch middle colic
123. diabetic footangio plus neuropathy
124. duodenal perforationgastroduodenal artery
125. patientthialand chills fever cause malaria?????+
126. urine leaking from umbilicus.patent allantoin
127. baro receptor respond to low bp...carotid sinus
128..actinmyocin clathrinpinocytosis
129. disc prolapsespinal nerve plus post root
130.amnioticmembnon immunogenic
131. last trimester circulation separated byendothelium + synctio
132. congenital cataract rubella
133.most common cuaaase of mental retardationfragile x syndrome..down was not in option
134..cvpinf,,,doc vanco
135- old man dental treatment hx of rhd doc clindaorvanco??+++
136. falciparum most impblack water fevere

34

1337..cerebral malariafalciparum
138.muscular dystrophy scanerio what will be raise cpk
139..inexcinc blood flow local metabolitr
140 heart blood flow -local metabolite
141.marathon race endhigh glucagon low insulin
142.mucous gand predominantlysublingual
143. creatitine produced by muscles
144. patient with uremic nnephropathynormocytic normochromic anemia
145. crf what to do fluid restriction to 0.5 l per day
146. nerve for tendon reflex correctly matched..tendon reflex
147..end arteries vital organ
148. tb diagnosis definitive..AFB
149. soft keratin ..stratiuncorneum
150 unconcious not responding to naloxonephenobarbitone
151..hyer ventilating patient with acidosis hx of drug intakesalicyclatepoisoining
152. souce of folate raw vegetables
153. definitive dx of folate defrbc folate level
154. mitral regurgitation..pan systolic ap apex
155.apex beat 8cm from midline
156. hep b and c spread contaiminated needles
157. common cause of pulmonary embolismthromboembolism
158. vma..pheochromocytoma
159..indol acetic acid ,,,carcinoid
160. lumber punctureabovr l4
161. some kind of like dec absorption requied when local effect tof drug is required
162. absorption in intestine inc byinc lipid solubility or ionization
163. counsellingto help people themselves
164. bucco pharyngeal membstomodium from foregut

35

165..dna containing organelle other than nucleusmitochondria


166. ciliamaximum mitochondria in apical portion
167..severe hypomagnesmiahypocalcemia
168.bio availability in oral route some thing like that
169. microscopic feature of malignancyinvasion
170. imp feature of malignancymetastasis
171..pulmonary vasoconstrictiondue to dec po2
172..fatty acid entersin kerb cycleacetyl coa
173. severe diarrhoea.metabolic acidosis with normal anion gap
174.chemotaxis most imp agent..c5a
175. HLA another q??
176. progesterone relation ALDOSTERONE, CORTISOL,CORTICOSTERONE
EDITED Dr M. Z. Babar

=-=-=-=-=-=-=-=-=-=-=-=-=-=-=-==-=-=-

36

(4) Medicine 11th May(Morning) by Usman Dar (192 MCQS)


11may2016morning Medicine&Allied
1.Length of trachea in neonat
a.4cm (ans)
b.6cm
c.8cm

2.After 48 hr in acute inflammation predominant cells- (neutrophils...(ans)

3.In Asthma fev<75(ans)

4.8 mnths old child with histoy of u.r.t.i n pharyngitis..lab shows dec
Leukocyte n dec no,of plasma cells X-linked A gammaglobiniemia(ans)

5.Hall mark of HIV- (proliferation of T-cells in side cell)ans

6.rectal temperature at which perminant cell death occurs


a.106 deg F ANS
B.104
C.No effect even above108

7.Inversion of foot- tibialis anterior and posterior(ans)

8.Carotid sheath
A.Behindtheinfthyroid
B.Behindphrenicnerve
C.Formed by investing layer(ans)

37

9.regarding posterior triangle


A.roof is formed by skin
B.base middle third of clavicle(ans)

10.true about neonate skull


A.vault is biggerthan face (ans)

11.neuro leptic malignant syndrome caused by drug


A.Haloperidol (ans)
B.reserpine

12.regarding adenosine
A.causes transient tachycardia(ans)
B.block both sa and avnode
C.half life 30 sec
D.theophyline enhances its activity

13.how sympathetic system effects carbohydrate metabolism


A.Epinephrine causes glycogenolysis(ans)

14.germ cell reach gonadal ridge at A.6 weeks(ans)

15..thrombo cytopenia caused by A.quinine B.chloramphenicol(ans)

16.mallorybodiesseenin A.alcoholicliverdisease(ans)

17.aclimitized person adequate oxygen supply by A.inc HB concentration(ans)


18.polydyctaly,microcephaly in which syndrome

38

A.edward(chromosome18)
B.patau(chromosome13)(ans)

19.vomiting centre in
A.medulla (ans)

20.S4 is due to
A.atrials ystole (ans)
B.ventricularfilling

21.chemo induced vomiting treatment


A.ondensetron(ans)

22.subclavian vein related with


A.1strib(ans)

23.at the end of ventricula rejection


A.pulm valve close (ans)
B.aortic valve remain open
C.av valve close

24.turner syndrome features were asked


A.gonadal dysgenesis(ans)

25.half life depends on


A.drug clearance(ans)
B.drug distribution

39

26.regarding warfarin
A.not given in pregnant women(ans)
B.immediatelyantagonisedbyFFP

27.regarding aphasia (poor recall bcq) A.dominant hemi sphere(ans)

28.abnormal lumbar vertebral curvature A.lordosis(ans)

29.vital capacity decreases


A.TLC decrease(imarked)
B.dec surface tension
C.paralysis of inspiratory muscles
D.paralysis of expiratory muscles

30.renal vein A.ant to aorta(ans)

31.a question was regarding nephrogenic diabetes insipidus.

32.autosomal dominant disorder A.heriditary spherocytosis(ans)

33.pelvic appendix scenerio

34.from.which area of brain cholinergic fibres of cerebral cortex mostly arise A.nucleus basalis of
mynert(ans)
35.hypoglycemic effect of sulphonylurea enhanced by A.phenyl butazone(ans)
36.sodium retention is due to A.heart failure(ans) B.excessADH

37.chorea regarding question A.basal ganglia(ans)

40

38.para sympathetic action A.erection(ans)

39.scenerio in which factor 8 was 85% factor 97%levellow.APTT prolonged A.christmas disease(ans)

40.patient with gum bleed,petechie,advised bone marrow biopsy A.leukemia(ans)


41.denial scenerio.

42.a female patient in came,her tone was seductive(pastbcq) A.call nurse(ans)

43.a child with dec platelet count,after acute viral infection


A.anti bodies to platelets(ans)
B.pooling in spleen
C.Dec bone marrow synthesis

44.osteogenesis imperfect scenerio A.dec collagen ya defective collagen(ans) Poor recall but this was the answer

45.before skilled voluntary movement by which area activate


A.cortical association(ans)
B.premotorarea
C.cerebellum

46.pleural tap done,specific gravity1.006 A.ccf(ans) B.TB C.pneumonia

47.basicdrugs bind to A.alphaglycoprotein(ans) B.albumin


48.premalignant lesion
A.compoundnevus

41

B.junctionalnevus
C.intradermal nevus(ans)
A&barebeningn
49.LMW H action was asked
A.acts on factor10.(ans.).
50.premalignant lesion of skin
A.bowendisease
B.actinic keratosis(ans)
51.infarctionposterior1/3 of inter ventricular septum
A.RCA(ans)
B.LAD
C.Marginalartery
52.which congenital anamoly associated with increase risk of cancer
A.bladder extrophy(ans)
53.right gastric artery branch of
A.Hepatic artery(ans)
54.tip of nose nerve supply
A.ophthalmic(ans)
55.regarding nucleolus
A.lack membrane(ans)
B.ribosomes+dna
56.cavernous sinus thrombosis A.sup ophthalmic vein(ans)
Note..cavernous sinus infection=>infophthalmicvein
57.IgA deficiency scenerio

58.giant cell arteritis scenerio

42

59.fibroblast feature was asked Forgotoptions:(

60.a question regarding skeletal muscle A.multiplenuclei B.

61.tumor extend to nerve sheath A.carcinomaex pleomorphic(ans)

62.mitochondria receive protein synthesis by


A.RER(ans)
B.ser
C.ribosomes

63.fibroblast release A.extracellular matrix(ans)

64.respiratory symptoms due to A.ascaris(ans)

65.mosquito bite=>filariasis

66.hypo physis cerebri related posteriorly A.sphenoid(ans)

67.outermost covering of nerve sheath A.epineurium(ans)


68.sensitive test for SLE A.ANA(ans)
69.75 teachers average BP was 80.standrad deviation 10,% age of
Patients having BP more than70 A.84%(ans)

70.Test to rule out disease A.specific(ans) B.sensitive C.accuracy

71.extra iron stored as(scenario shows pathological storage) A.hemosiderin(ans)

43

72.true about thyroid gland A.veins drain into internal jugular & brachio cephalic vein(ans)

73.parotid duct opening A.upper 2nd molar tooth(ans)

74.True about cell membrane A.allow glucose to pass in presence of insulin(ans)

75.vitamin helps in clotting factor synthesis


A.vitC(ans)
B.vitb12
C.vitE
D.vitb6

76.stylo pharyngeus muscle derived from


A.2nd arch
B.3rd arch(ans)

77.lacrimation after food in take due to damage of


A.facial nerve(ans)
B.lacrimal.nerve
C.trigeminalnerve
Note..als ocalled Crocodile tears

78.boy with recurrent respiratoryn GI infections lab shows normal


leukocytes,,absence of plasma cells cause
A.IgA deficiency
B.CVID

44

C.complement deficiency
D.x-linkeda gamma globinemia(ans)

79.after 48 hrs acute inflammation predominant cells


A.neutrophils(ans)
B.macrophages
C.lymphocytes

8o.testicular feminization syndrome typical scenerio

81.phrenic nerve relation was asked A.superficial to scalenus anterior muscle(ans) This was best option

82.aortic dilatation at aortic opening in diaphragm compresses A.azygous vein & thoracicduct(ans)

83.turner syndrome A.gonadal dysgenesis(ans)

84.prevent progression of atheroma A.atorvastatin(ans)

85.fastest fibres Ans.purkinjefibres

86.pulmonary embolism cause Ans.DVt

87.ascenerio was of randomizedcontrol trial

88.pulmonary embolism goes to most probably Ans.pulmonary artery

89.byssinosis A.textile industry(ans)

45

90.strontium causes Ans.osteosarcoma

91.a question regarding Virchow triad.

92.aplasia Ans.dec production of cells

93.microscopicfeatureofantpituitary Ans.smallest chromophobes

94.secossificationcentreafterbirth Ans.distal end of femur

95.loss of sensation of medial1/3 of dorsal and palmar side due to damage of Ans.ulnar nerve

96.old lady good pasture syndrome BP always raised not taking Medication properly,died,on autopsy inc heart siz e
will be due to Ans.hypertrophy

97.ossification centre at 11yrs old girl Ans.lateral epicondyle

98.testosterone action was asked


A.laryngealhypertrophy(ans)??
B.pubichairincconcavity
C.darkeningofareola

99.slowest growing thyroid CA Ans.papillary carcinoma

th

100.known case of lumbago tender4,5 intercostals space,chest pain,cardiac enzymes normal Ans.chosto
chondritis

46

101.lightreflex Ans.2&3nerve

102.lung CA case,hemoptysia due to Ans.bronchial artery erosion

103.verticalsectionofopticchiasm Ans.bitemporalhemianopia

104.aorticdissection Ans.medialnecrosis

105.abscess caused by staph aureus, spleen shows wedge shaped necrosis Ans.coagulative necrosis

106.Staph aureus,lab diagnosis Ans.coagulasepositive

107.Aquestion was Regarding digoxin toxicity Ans.potassium loosing diureticuse

108.vomiting centre in Ans.medulla

109.drug used in vomiting after chemotherapy Ans.ondansetron

110.female taking steroids,her wound is not healing due to


A.decreased collagen formation (ans) B.dec neutrophil migration

111.severe hypercalcemia treated with Ans.loopdiuretics

112.drug causing histamine release Ans.morphine

113.intruepelvis Ans.middle rectalartery


114.esophageal phase of swallowing is effected in

47

A.scleroderma(ans)
B.myastheniagravis
C.polio

115.one question was cardio vascular reflex organization maintains


A.arterial blood pressure(ans)
B.CO
C.arteriolar resistance
D.oxygenation to tissues

116.lower motor neuron Ans.muscle wasting

117.UMN Ans.increased muscle tone

118.hemmorhagic infarction occur in


A.lung(ans)
B.spleen
C.heart
D.kidney
Note..hemorhagic or red infarctinlungan dgit
Pale infarct or anemic infarctinHSK
H=heartS=spleenK=kidney

119.which organ blood supply least affected by exercise


A.brain(ans)
B.skin
C.skeletalmuscle

48

D.kidney

120.rapid aclimitization causes Ans.pulmonaryedema

121.adult daily protein requiremnt Ans.50gm

122.regarding thymus Ans.rich n lymphocytes

123.raynaud'sphenomenon Ans.peripheral vascular disease

124.prolonged PR interval,QRSafter3Pwaves3:1ratio,typeofblock ans.mobitztype2

125.Thorn prick abcess due to Ans.Staphaureus

126.inc venous return dueto Ans.calf muscle contraction

127.shortPRinterval Ans.WPW

128.HB starts to appear in which st age of erythropoiesis


A.proerythrocyte.
B.reticulocyte
C.earlynorrmoblast
D.pronormoblast
129.estrogen containingOCP's causes Ans.DVT

130.in MRSA resistence which drug is given Ans.cotrimaxazole

49

131.gonococcus diagnosis Ans.gram staining

132.vit D levelassessedin lab Ans.25 OH cholicalci ferol

133.nucleus tractus solitarius Ans.taste fibres

134.fragilex-syndrome Ans.Tri neuleotide reapeat shift

135.aldosterone cause A.Nareabsorption(ans)

136.ADH stimulate by Ans.dec plasma.Volume

137.good pasure syndrome Ans.linear depositson IF

138.Chronic pancreatitis,steatorrhea,enzyme deficiency Ans.lipase

139.RTA head injury symptoms of Central diabetes inspidus Ans.Central diabetes insipidus

140.in an ion gap Na is multiplied by 2other =ans.Anions

141.leukocyte adhesion to endothelial cell Ans.LFA-1

142.DIC=thromboplastin

143.reducedlymphaticflow Ans.exercise

144.angiotensin cause Ans.vasoconstriction

50

145.cck inhibits Ans.gastric emptying

146.gastric secretion decreased by Ans.secretin

147.free ribosomes Ans.RER

148.diabetic patient with vision problem.visual acuity is 3/60&4/60. Ans.cataract

149.boat shaped cyst Ans.pneumocystisjeroveci

150.least AV difference Ans.Heart

151.Estrogen secreted by Ans.ovaries

152.perianal lesion Ans.crohn'sdisease

153.strontium -90.Incorporated in food chain Ans.osteosarcoma

154.BBB crossex easily by Ans.CO2

155.farmer,organo phoshphate poisoning scenerio Drug willb given Ans.atropine

156.patient on ATT, uricacid level high,due to which drug Ans.pyrazinamide


157.obliteration of uterus in pregnancy due to
A.parietalis&basalis
B.parietalis&basalis
C.amnion&chorion

51

158.thiazidesideeffect Ans.hypokalemia Note,hyperuricemia was not in option

159.bile salts absorbed Ans.ileum

160.muscle paralysis caused by drug Ans.Curare

161.undescended testes risk factor for Ans.malignancy

162.tetanus=toxoid

163.wilson disease scenario diagnosis Ans.serum cerulo plasminlevel (rabiabcq)

164.tumor suppressorgene Ans.P53

165.extreme left shift of oxy-HBcurve A.COpoisoning(ans) B.HbF

166.highest serum conc Ans.IgG

167.one question regarding clonus

168.burning Hot temp on skin Ans.45c

169.EEG waves closed eyes Ans.alpha waves

170.TBM scenerio

171.patient on dialysis,anemic Ans.dec erythropoitin

52

172.Hall mark of HIv Ans.proliferation of virusint-lymphocytes

173.CD4.=helperTcells

174.Adenoma benign change most likely shows Ans.glandular pattern

175.muscle which open mouth Ans.lateral pterygoid

176.pulm congestion=heartfailure

177.iron deficiency anemia scenerio

178.RMP maintained by Ans.Na/Kpump

179.diphtheria severe effecton Ans.myocardium

180.dorsal rami Ans.extensorsofback

181.Growth hormone secretion Ans.exercis

182.epinephrine causes vasodilationby Ans.Beta2receptors

183.carotid sheath formed by Ans.investing layer ofdeep cervical fascia of neck

184.subclavi n vessels groove over Ans.first rib

53

185.gentamicin Ans.nephrotoxic

186.in third degree heart block Ans.syncope due to delayed a nd irregular ventricular contraction

187.in organic compound associated with malignancy Ans.asbestos

188.won't need artificial breathing if lesion is Ans.belowC5

189.true about facial nerve Ans.post belly of digastrics supplied by facial nerve

190.fibroblast ans.amorphous ECM synthesis

191.True regarding heart Ans.Sa node in sub epicardium in right atrium


Others made no sense

192.rhythmic muscle contraction after stretching Ans.clonus

=-=-=-=-=-=-=-=-=-=-=-=-

54

(5)Surgery 9th May (Evening ) by Syed Arsalan Akbar (186)


PAPER SURGERY 9TH MAY 2016 EVENING -DR SYED ARSALAN AKBAR GILLANI (SMC SWAT)
NOTE..TOTAL QUESTIONS 186...KINDLY DO SHARE UR PAPERS WHEN U PASS.AS IT IS A CHAIN THAT WE
SHUD NOT BREAK...

A medical student was studying a microscope slide of transitional epithelium. He noticed


unduly thick and darkly stained luminal plasma membrane of the surface cells. It is most likely
due to
A. an artefect
B. a layer of glycocalyx
C. aggregation of cytoskelatel elements
D. oblique section of plasma membrane
E. presence of surface microvilli

2. The root value of nerve supply to all short muscles of hand is


A.c6
B.c7
c. T2
d. T1
e. T4
3.What type of tissue is critical for dividing the heart into four chambers and the outflow tract
into pulmonary and aortic channels
A. aorticopulmonary septum
B. endocardial cushions
C .conus septum
D. septum primum
E. septum secondum

4.A person developed urine infection.Urine culture plate yielded a greenish growth having
fruity smell.This organism was oxidase positive and non lactose fermenting. The most
appropriate drug in this case would be
A. Ampilcilin
B. Ceftazidime
C .Ceftriaxone
D. Ciprofloxacine
E. Gentamycin

5. The surface marking of the oblique fissure of lung would be


A. From T2 spine around thorax to 5th costochondral junction
B. From T2 spine around thorax to 6th costochondral junction

55

C. From T3 spine around thorax to 6th costochondral junction


D. from T4 spine around thorax to 6th costochondral junction
6. The structure most likely to be effected by pus in the adductor canal is

A. femoral vein
B. great saphenous vein
C. medial circumflex vein
E. profunda femoris vein

7. Pain impulses originating in peptic ulcer are carried by


A. 7th intercostal nerve
B. 8th intercostals nerve
C. greater splanchnic nerve
D lesser splanchnic nerve

8.To auscultate the tricuspid valve , stethoscope should be placed at the


A .left 4th intercostals space in mid clavicular line
B .left 5th intercostals space in mid clavicular line
C .medial end of 2 nd left inetrcostal space
D. medial end of 2 nd right intercostal space
E. Right half of lower end of body of sternum

9. Loss of flexion of knee and extension of hip are caused by paralysis of


A. Rectus femoris
B Sartorius
C. Semitendinous
D. Vastus lateralis
10.Absorbtion of fluid from intestinal epithelium depends on
A. glucose and sodium in lumen combine.??????
B paracellular junction

11. Occlusion of RCA after giving marginal branch which of heart is affected
A. SA node
B. AV node
C. Right atrium
12. Lower lip bleeding on left side pressing on which artery would stop bleeding
A. angle of mouth
B, left facial artery
C. facial artery on both side
13. Osteoblastoma most common site is
A. epiphysis
B. Metaphysis
C. vertebral body

56

D. diaphysis
14.Nulliparous women with hair growth on face lips and oligomenhorea

B. polcystic ovarian disease


c. pituitary tumor

15. Aldosterone secretion increased by


A. hypokalemia
B. ACE inhibitors
C. dec blood volume
16.A pregnant lady having reheumatic heart disease.Its characteristics valvular lesion will be
A.Mitral stenosis
B, Mitral regurgitation
C. Aotic stenosis
17.Pateint is having myocardial infarction. Posterior 1/3rd on interventricular septum is
involved.which artery is affected
A.right coronary artery
B. left coronary artery
C. anterior descending artery
D. right marginal artery
18.Anatomical structural unit of heart which characterize conduction of heart is
A .Intercalated disc??
B. Gape junction. ??????
C. Tight junction
D. Desmosomes
E. Zonula adherens
19. malignancy associated with HIV/Aids
A .leukemia
B. Kaposi sarcoma
20. During mitosis chromosomes replicate in which phase
A. prophase
B. Metaphase
C, anaphase
D. Telophase
E. Interphase
21, Type of sampling which every one has equal chance of being selected
A. systemic sampling
B. Random sampling
C. case control
D. biased sampling

57

22.During exercise increased blood supply to muscle is via


A. local metabolites
23.Difference between systemic and pulmonary circulation is
A. more blood flow in pulmonary circulation
B. low resistance in pulmonary circulation
C.high resistance in pulmonary circulation
24.Chemotherapy given and tumor size become small this mechanism is by
A. atrophy
B, necrosis
C. apoptosis
C. hypertrophy
25 Columnar epithelium lines
A. Rete testis
B. germinal layer of ovaries
C. pigment epithelium of retina
D. Uterine tube
E. Epitheloid covering of choroids pluxes
26. Clavipectoral fascia
A. invest pectoral major muscle
B. invest pectoral minor muscle
27. Structure pass through greater sciatic foramen
A. Sciatic nerve
B. obtuarter inrernus
28.Which one of the following is equal under normal physiological condition
A. coronary and cardiac output
B. Cardiac output and muscle blood flow
C. Venous return and pulmonary blood flow

29. Anterior and posterior Humeral circumflex arteries are branches of


A. Brachial artery
B. Axillary artery
C, subclavian artery
D. thyrocervical artery

30. Dorsal scapular nerve supply


A. Latissimus dorsi
B. Levator scapulae

31. Man with history of 40 pack per day smoking developed carcinoma brain which was
secondary..primary carcinoma is located in

58

A. Lungs. confrm it
B. Bladder
C. Liver
D, stomach
32. Occiptal nerve supply
A. rectus capitas anterior
B. rectus capitus posterior

33.A 20 year old female presented with right sided cervical lymphadenopathy. She was
completely asymptomatic .Histological examination reveal a metastatic tumor. The most likely
primary carcinoma thyroid in this patient is
A. Anaplastic
B. follicular
C. Medullary
D. Papillary

34..A male patient after a few days of severe burns developed high grade fever .Blood culture
yielded of gram negative rod. He suddenly developed a bleeding diathesis. The most likely
cause in this setting is
A. Drug induced aplastic anemia
B. Hepatorenal syndrome
C. thromobocytopenia
D. Liver failure due to septicemia
E. Disseminated intravascular coagulation

35. Origin of peroxisome is from


A. rough endoplasmic reticulum
B. Golgi apparatus
C, smooth endoplasmic reticulum
D. nucleolus

36.A farmer presented with linear migratory lesion on his foot. Which parasitic infection is
most likely
A. cutanous leishmaniasis
B. hook worm
C. cutanous larva migrans
D. strongyloids stercolaris
E, dracanculus medinensis

59

37.A

55 year old non insulin dependent diabetic female had major abdominal
surgery. On the 8th post operative day she complained of suddent onset of
dyspnea, chest pain and haemoptysis. The most likely cause of her symptoms
would be
A. Mycocardial infarction
B. pneumonia
C. pulmonary embolism
D. septicemia
E. spontaneous pneumothorax

38.An adult alcoholic male complained of dyspepsia and loss of appetite . Liver biopsy revealed
Alcoholic liver disease because of following pathological microscopic feature
A. extensive hepatic fibrosis
B. fatty change
C. Mallory bodies
D. nutritional disturbance
E. proliferation of bile ducts

39.Increased GFR and increased blood flow occur due to


A. dilatation of efferent arteriole
B, dilatation of afferent arteriole

40.Amniocentesis is done
A. after 2 weeks
B. after 14 weeks
C. after 4 weeks

41.Ist response against acute inflammation in tissue is


A. neutrophill
B. macrophage
C.lymphocytes

42. Pulmonary artery supply


A. primary bronchi
B. secondary bronchi
C. Tertiary bronchi
D. Alveoli

60

43. Most common fracture of long bone in lower limb is


A.Tibia
B. femure
C. humerus

44. 1 gm of protein gives energy


A. 2 Kcalories
B. 4 Kcalories
C.8 k calories
45.A 2 year old boy has been brought to the hospital for circumcision. There is history of excessive
bleeding from the umbilical stump at the time of birth. Two cousin bleed profusely after
circumcision by surgeon. The blood test most likely to give a clue to the diagnosis is
A. platelet count
B. Bleeding time
C. clotting time
D.prothrombin time
E. Activated partial thromboplastin time

46. Edema is caused by


A. decreased sodium level in serum
B. Haemoconcentration
C. Increased hydrostatic pressure
D. increased oncotic pressure
E. increased venous return

47. An antigen found in relatively high concentration in the plasma of normal


fetuses and a high proportion of patients with progressive carcinoma of the
colon is most likely
A. Alpha fetoprotein
B. Alpha interferon
C. Carcinoembryonic antigen

61

D. heterophil antigen
E. viral antigen

48.Which one of the following is the primary opsonin in the complement


system
A. C1 q
B. C3a
C. C3b
D. factor B
E. c5

49.Teratomas are tumor composed of


A. All types of ectodermal tissue
B. Mainly cartilage and epithelial cells
C. Derivatives of all three germ layers

50. Edema is caused by


A. decreased hydrostatic pressure
B. Increased osmotic pressure
C. decreased permeability
D. lymphatic bloackage

51.Man is intermediate host in


A. filariasis
B. Hook worm infection
C. Hydatid disease
D. trichomoniasis

52. Right supra renal vein drain into.IVC

62

53. Metaplasia is characterized by


A. Variation in size and shape of cells
B.. Cloudy swelling of cells
C. A functional change in cells
D. an increase in the number and size of cells

54. P 53 is tumor suppressor gene, When it is absent or mutated it indicate


A. apoptosis
B. Cell survival
C. DNA repair
D. Mitosis

55. The edema fluid in acute inflammation


A. is known as transudate
B. has specific gravity 1010
C. has protein more than 3 gm /dl
D. does not contain fibrongen

56. Female with blood group A have 2 children, one with O and other with
AB, the blood group of father is
A. A group
B. O group
C. Two childrens have two fathers
D. B group

63

57. Most common cause of multiple fracture in adult is


A. cushing syndrome
B. vitamine D defeciency
C. Osteoporosis
D. Hyper parathyroidism

58.Protrusion of mandible occur by which muscle


A. medial pterygoid
B. buccinator
C. lateral pterygoid

59. Fuction of notochord


A. give rise to CNS
B. develop into spinal cord
C. Induction of ectodermal neural tissue to produce neural plate

60. Release of neurotransmitter is due to


A. release of calcium into presynptic terminal

61.Straight vessles which lie along side the loop of henle particularly in the
juxa medullary nephron and are most imp for concentrating of urine are
A. arcuate arteries
B. inter lobular arteries
C. Peritubular capillary network
D. vasa recta

62. During pharyngeal stage of swallowing


A. epiglottis remain stationary
B. soft palate moves downward
C. vocal cards are abducted
D. larynx moves up and forward
E. palatopharyngeal fold move apart

63. The most important buffering system in the body is


A. Hco3
B, hemoglobin
C. phosphate
D. plasma protein

64.Right border of heart on xray also visible apart of


A. SVC
B. IVC.

64

C. left atrium

65. During inspiration what decreases


A. heart rate
B. Venous return
C ,negative intrapleural pressure note: negative thoracic pressure increases
D. arterial pressure
E.volume of pulse pressure

66.Most common cause of pulmonary embolism is


A. Deep venous thrombosis
B. left ventricular myxoma
C. abnormal pulmonary blood flow

67.typhoid fever, test during 1st week ix


A. blood culture
B. urine culture
C. stool culture

68. Typhoid fever during 2nd week ix


A. blood culture
B. blood culture and widal test
C. urine culture
D. stool culture

69. Benign neoplasm is


A. adenoma
B. heapatoma

70.which of the following structure lie in right free edge of lesser omentum
A. common bile duct,cystic duct and hepatic artery
B. hepatic vein and cystic duct
C. portal vein, common bile duct and hepatic artery
D. portal vein, hepatic artery and hepatic vein

65

71. Diabetic nephropathy investigation of choice


A. Ultrasound abdomen
B. urine albumin
C. serum creatinine clearance

72. Mean systemic filling pressure is regulated by


A. heart rate
B. venous return

73. origin of oxytocin and ADH is


A. post pituitary gland
B. Hypothalamus

74. A patient with history hemorrhage receives blood stored for 2 weeks
mainly contain
A. platelets
B. fresh frozen plasma
C, RBC

75. stress hormone in our body is


A. epinephrine
B. insulin
C. ACTH

66

76. A young patient with serum creatanin of 8 mg % presents with a blood


pressure of 210/130 mmhg. The structure most likely damaged is
A. cells in macula densa
B. cells in peritubular capillaries
C. cells in renal glomeruli
D. Juxta glomerulae cells

77. The sciatic nerve enter the gluteal region through foramen
A. Greater sciatic
B. lesser sciatic

78. Laborotory diagnosis of leprosy in its early stage is by


A. Bacterial culture of blood
B. Bacterial culture of exudates
C. sputum scraping for leprae bacilii
D. Nasal scrapings for leprae bacilli
E. Mantoux test

79.A 28 year old male with history of jaundice and raised ALT. The liver
histology shows councilman bodies. These are most likely to be process of
A. coagulation necrosis
B. Apoptosis
C. enzymatic fat necrosis

80.A 20 year old female presents with cervical lymphadenopathy. The


histological examination of lymph node showed granulomata. The diagnosis of
tuberculosis in this case is strongly suggested due to
A. Epitheloid cells

67

B. Giant cells
C. Mantoux Test
D. Caseous necrosis

81.A 28 year Fisherman presented with slowly developing lethargy, easy


fatigue and palpitation. His diet comprises of fish and rise. On physical
examination He has pallor ,loss of touch sense. His CBC showed HB 7,5
platelet 110000.His stool report showed Ova of intestinal parasite. The most
likely causative parasite is
A. Ankylostoma duodenale
B. Ascaris lumbricoides
C. Diphylobothrium Latum
D. Taenia saginata

82.A man of 40 year old developed bilateral enlargement of cervical lymph


node. The excised lymph node biopsy revealed Hodgkins disease. Which one
of the following type has best prognosis?
A. lymphocyte depletion
B. mixed type
C. Lymphocyte predominat
D. Nodular sclerosis

83.GVE fibre of vagus arise from


A. nucleus ambiguous
B. Dorsal nucleus

84. Female with infection of HPV comes after 2 years pap smear shows
prominent nuclei and increased nucleus size. It is
A. Metaplasia
B. Dysplasia
C. anaplasia

68

85.Mucle of back innervated by


A. dorsal ganglion
B. Dorsal rami

86.Gamma efferent supply to


A. extrafusal fibre
B. end of intrafusal fibre
c. central of intrafusal fibre

87.Child with yellow sclera and colored urine best investigation would be
A. bilirubin
B. bilirubin plus ALT
C. AST

88. Standard deviation shows


A. size of sample
B. Reliability of sample
C. Variability among individual
D. Validity of conclusion
E. Quality of experimental design

89.Potassium is mainly regulated by


A.Angiotensin
B. epinephrine
C. Aldosterone
D. sodium

90.True hermphrodiate is

69

A. xxy
B. xx..its controversial question..xx ratio is more in true hermophrodiate(54%)..some ppl say
xxy is cpsp demo qstion..but no one has ever seen this demo questionxx/xy is asim and shoib
gynae keychoose between xx or xx/xynever choose xxy
C. xx/xy
D.xyy

91.Phosphorylase enzyme raised to cause gylcogenolysis due to deficiency of


which hormone
A. cortisole
B. glucagon
C. Insuline
D. Epinephrine

92. Discharge from muscle spindle decreases when


A. Muscle relaxes
B. Muscle contract

93. Large increase in blood volume occur in


A. Hypotonic Nacl
B. Hypetonic Nacl
C. distilled water

94. Nerve accompany superficial temporal artery is


A. Auricular temporal nerve
B. cervical nerve

70

95. Isotonic and isometric contraction. Isotonic is different by


A. consume more phosphate bonds
B. less work load
C. have more energy of recovery
D. Heat production

96. Patient with tachycardia and palpitation, TSH is low but on giving TRH,
TSH is increased. problem is in
A.primary hypothalamus
B. anterior pituitary
C. Thyroid gland

97. Myocardial infarction due to blockage of left circumflex artery which will
be affected
A.left ventricle and right ventricle
B. Left atrium and left ventricle

99.Highest cholesterol content is in


A. Egg
B. white meat
C. Red meat

100. Tip of scapula at body level of


A. T 6 body
B. T7 body
C. T8 bodyhere body is mention..never choose T7 body..choose t 7 spine or T8 body

101. About Collagen fibre


A. Abundant in ligamentum flavum

71

B. stained with Eosin dye


C.Have elastic recoil
D.Branching fibre

102. During procedure of Esophagectomy, the stomach is mobilized upto


esophagus, which artery will responsible for blood supply of stomach
A. Right gastric artery
B.Left gastric artery
C. right gastric epiploic artery
D. Lest gastric epiploic artery

103. S2 heart sound is produced by


A. closure of atrioventricular valve
B. closure of aortic and pulmonary valve
C. Ventricular filling
D. Atrial systole

104.Protein content less in glomerular filtrate occur due to


A. pore size
B, Pore size and negative charge

105. Man Got gun injury lateral side of upper leg, He is unable to dorsiflex
because of
A. Fracture head fibula
B. Fracture neck of fibula
C. Fracture of shaft of fibula

106. The most common site for amoebic ulcer in the gut is
A. Ascending colon
B. Caecum

72

C. Rectum
D.sigmoid colon
E.Terminal ileum
D. Fracture of condyl of tibia

107. Adult polycystic kidney disease is most closely associated with


A. cerebral hemorrhageasim key
B. hemoptysis
C. polycythemia
D. renal failure

108. The maximum effect achieved by a drug is measure of its


A. bio availibilty
B. potency
C. safety
D. efficacy
E. intrinsic activity

109. A 20 year old male has dyspnea on lying down. The structure most likely
to cause this is
A. Enalrged ishmus of thyroid
B. multinodular goiter
C. puberty goiter
D. retrosternal goiter

110. A 50 year old has hysterectomy for uterine myxoma.then she develop
lump in lower abdomen after 6 month.It was diagnosed as a case of incisional
hernia. The ist structure this lump is most likely to push from with in out
ward is
A.internal oblique muscle

73

B. transversalis muscle and transversalis fascia


C. Transversalis fascia
D. transversalis muscle

111.Whole wheat is an excellent source of


A. ascorbic acid
B.riboflavine
C. thiamine
D. Vitamina A
E. Vitamine D

112.Foreign body most commonly lodge in


A. left inferior lobe
B. right inf lobe
C. carina

113. Left gastric epiploic artery is branch of


A. hepatic artery
B. spleenic artery
C. gastrodudenal artery

114. gaseous exchange occur where


A. simple squamous epithelial layer

115, Most sensitive of cardiac tissue is


A. Ckmb
B. Troponin T
C. Myoglobin

74

116. epinephrine released from adrenal medulla causes vasodialation by


which receptor
A B1
B alpha 1
C b2
D alpha 2

117. A girl of 8 years at terminal illness is asking to you Am I going to die


you response shoul be
A. whats your parients have told you

118. counseling is A. helping patient themselves


119.Patient injured in accident multiple fracture bo 90/60 pulse 120
A. fluid replacement

120. Anemic hypoxia is caused by


A. cyanide posining
B. methehemoglobinemia

121. Amoebic liver abcesss diagnosis is


A. serology

122. characteristics finding of clostridium botulinum is


A. flaccid paralysis of respiratory muscle
B. diarrhea

123. phase 1 biotransformation reaction


A. oxidation
B. reduction

124.. natural defence against cancer is

75

A. apoptosis
B, necrosis

125,Pateint has proteinuria greater than 6gm /dl structure demaged


A. Pct
B. Dct
C. Basement membrane

126. cells which contain sense granule and have ig E antibodies on their cell
membrane are
A. esonophills
B. macrophage
C, Basophills

127. Lymphatic drainage from nipple of breast is


A. central axillary lymph node
B. ant axillary lymph node
C. thoracic lymph node

128. Pain mediator is


A. bradykinin
B. histamine
C. serotonine
129. Adductor canal is formed by
A. Sartorius and femur????
B. vastus mediallis and adductor comp???
C. flxor and adductor muscle

130. nerve supply of geniohyoid


A. genioglossus
B. C1

76

C. anca cervicalis

131..Structure passing above piriformis muscle injured .most likely it will be


A. sciatic nerve
B. obturator nerve
C. superior gluteal vessels

132. Vagus nerve passes through


A. foramen magnum
B. jugular foramen

133. Calcitonin is tumor marker of


A. thyroid glanad
B. parathyroid gland

134 Bar area of liver limited by


A. Right and left triangular ligamnents
B. Coronary ligament
C. Falciform ligament

135.down syndrome
A. trisomy 13
B. trisomy 18
C, trisomy 21
D. monosomy 21

136.excreation of pencillin is inhbited by


A. gentamycine
B.probanecid
C.rifampine

137.Hiv after surgery cleaning of room is done by

77

A. 2% glutaraldehyde
B. 4% glutataldehyde

138. 5th posterior intercostals artery is branch of


A. thoracic aorta
B. Internal thoracic artery

139.Ureter is most constricted at


A. sacral promontry
B.sacroilliac joint
C. Transverse process
D. ischial spine

140. arterior tibial syndrome which artery is involved


A. anterior tibial artery??????
B.popliteal artery
C. dorsalis padis artery

141.Nucleus gracillis and cunateous lesion


A. Anaesthesia
B. Analgesia
C. Asterognisia

142. Cerebellar lesion causes


A. resting tremor
B. dytonia
C. Dysmetria

143.True regarding brain blood supply in brain


A. Middle cerebral artery supplies foot area

78

B. Arteries dont anastomose on surface of the brain


C. Arteries dont anastomose after entering the brain substance
D. Internal capsule is supplied by anterior cerebral artery

144. From 2 month to 6 year meningititis is caused by


A. streptococus pneumonia????
B. Nessiria meningitides
C. Hemophillia influenza

145.Usual haemoglobinopathy defect in


A. heme
B. in heme globin binding
C.. Globin structure defect
D. Amino acid sequence change in Globin????

146.ist pass metabolism ..ORAL


147. Inhalational agent which increases cerebral blood flow
A. Halothane
B, Isoflurane
D.sevoflurane????

148. Man moving down is elevator


A. Utricle
B. saccule
C. semicircular canal

149. Main support of uterus which held in position


A. Transverse cervical ligament
B. Broad ligament

79

C. Round ligament

150.Hypercalcemia activated
A. 1,25 dihydroxycalciferol
B. 24,25 Di hydroxycalciferol
C.25 hydroxy calciferol

151. Correct about thyroid gland gland


A.lymph drainage to deep cervical lymphnode

152. cuttin of parasympathetic system mostly


A. affect pulmonary resistance
B affect sphincteric action
A. affect GI muscles

153.Orarganelle where protein combine with carbohydrate, packed and


release
A.Agranular endoplasmic reticulum
B. Granular endoplasmic reticulum
C. Gogi apparatus
D. Ribosomes

154.Correct about DNA


A. DNA is single stranded
B.Euchromatin is transcriptionally active

155.Cause of delay in healing


A. infection

156.Pao2 decreased,Pco2 increased. Hydrogen ion increased manifestation is


A. hyperventilation
B. Hypoventillation

80

157. Thorn pick in left lower limb caused abscess


A. Staph aureus

158.Philidelphia chromosome present in


A. CML

159.Giant cell tumor histological finding


A. Soap bubble appearance

160.estrogen containing OCP causes


A. breast carcinoma
B. cervical cancer
C. Thromboembolism

161, plasmodium falciparum finding


A.black water fever
162 massive spleenomegaly,fluid thrill
A. infectous mononucleosis
B.sickle cell anemia
C.myelofibrosis

163. Carcinoid syndrome is diagnosed by


A. VMA in urine
B. 5 hiAA(hrdoxy indole acetic acid)

164,Brain stem stroke vertebral artery dissection having dysphagia cause


A. nucleus ambiguous
B. corticocerebellar tract
C. nucleus solitarus
D. inf cerebellar peduncle

165. about cubital fossa

81

A.scaphoid form floor of it


B. radial artery is present in its roof
C. cephalic vein arises from its roof

166.Rheumatic fever and infective endocarditis treatment to prevent disease is


A. pencillin G
B. Vancomycin

167. About autonomic nervous system


A. parasympathetic increase salivary secretion
B. parasympathetic increase sweet gland secretion

168. PTU action


A. stop thyroid hormone secretion

169. Patient in semiconscious condition csf shows glucose was 20 mg , protein


was 150mg 20% lymphocytes diag
A. acute bacterial meningitis???
B. partially treated bacterial meningitis????
C. fungal meningitis
D. viral ..

170. estrogen effect in adult female


A. thick cervical mucus
B.. development of secondary sexual character
C. enlargement of breast

171.pulonary vasoconstriction occur due to


A. dec Po2
B. dec pco2
C. increase Po2

82

172. ulcer on chin and oral mucosa by


A. HCV
B.HPV???
C. EBV
D. CMV

173.Man with discharging sinus on face with yellow granules


A.streptococus
B. staphylococcus
C. Actinomycosis

174. Gram negative bacteroid fragillis


A.foul smelling

175.female with femur neck fracture swelling and redness of limb next what
will happen
A.Dic
B. pulmonary thromboembolism
176. women with involuntary movement and space occupying lesion in caudate nucleus.which
is most common clinical featurebindicating this lesion
A. intentional tremor
B.resting tremors
C. chorea
D.hemiplagia
E. tonic clonic seizure

177. which one of the following increases pancreatic hco3 secretion but not
pancreatic enzyme

83

A. glucagon
B, cck
C,secretin

178. A smoker has dyspnea has ph 7.4 pco2 31mmhg and hco3 19 mmol/l
diagnosis is
A. mixed respiratory alkalosis and metabolic alkalosis
B. compensated respiratory Alkalosis

179. Ischemia Of superior mesenteric artery involve


A. Duodenujejunal junction to 1/3rd of transverse colon
B. jujenum,ileum,cecum,ascending colon and 2/3rd tranverse colon
C. Duodenujejunal junction to 2/3rd of transverse colon
c

180.. Narrowest part of air way in neonates


A. Thyroid
B. Trachea
C. cricoid
D. Bronchi

84

181. most common cause of increase BT is


A. von villibrand disease
B.haemophillia
C. aspirin toxicity

182. A young soldier is hit by a sharp splinter which cut through femoral
artery in his left thigh. He bleed profusely from wound and is brought to a
hospital in state of shock.The most appropriate blood product for immediate
transfusion is
A. 20 days old whole blood
B. fresh frozen plasma
C. platelete concentrate
D. whole blood

183. Critical type of tactile signal are transmitted through tract located
A. in posterior horn of spinal cord
B. dorsal white colum of spinal cord
C. Anterior white column of spinal cord

184. Myxoid degeneration is most likely associated with


A. Carcinoid valve disease
B. infectous endocarditis
C. Libman sacks endocarditis
D. Mitral valve prolapsed

85

185. A young man presented with low grade fever for last three months. On
examination his nasal bridge was distorted and cervical lymph node were
palpable.Histology of these lymph node revealed non caseating granuloma
and positive AFB stain. Most probable diagnosis is
A. Tuberculosis
B. syphilis
C. Leprosy
D. cat scratch disease

186. A19 year old male with a petechial rash other wise completely well. There
is no history of bleeding, recent illness or use any medication . An examination
of peripheral smear is normal except for virtual absence of platelets. The most
likely diagnosis in this patient is
A. Acute lymphocytic leukemia
B. Drug induce thrombocytopenia
C. Idiopathic thrombocytopenia purpura
D. Iron deficiency anemia
E, pernicious anemia
=-=-=-=-=-

86

(6)Surgery 10th May (Morning) by Zia Ur Rehman (71 MCQS)


surgery paper b.......morning may 2016 0nline 10-05-2016

1- a pt with blood group a+ . his serum was not crossmatched with all the donors. which was inc
in hiz serum (G)
a) ig b) igM c)anti kelly d)duffy

2.- a pt with obstructive azoo spermia due to ....a). igA B)igm c)igg d)igD c)igE

3. most common in pregnancy


a] hypoalbuminemia b] hyperglycemia fasting c] inc uric acid d] inc urea

4-common site for oral melanoma


a] soft palate b]hard palate c] floor of mouth d]tongue e] alveoli

5- autosomal dominant
a] FAP b]thalasemia c] sickel anemia d] hemophilia d} ......

6- left atrium is supplied by


a. LCA b] RCA C] LCX D] LAD E]PDA

7- A Pt with massive hemoptysis, embolization of rt lung , intercostobrochonchal trunk(gGGGG)


a] ant to thoracic aorta b] arch of aorta c] anterolateral to des thoracic aorta d0 post lateral to desc
thoracic aorta e] post to thoracia aorta

8- actinomycosis
a] abscess b] abscess and ganuloma c] granuloma

9- infection and transfusion and antibody kuch tha dont remember

10- a 20 years old woman with bleeding per vagina high grade fever and inc pt aptt
A] dic b. henocshonlein purpura c. itp d. dic

87

11- primary hyperparathyroidism a] post menupausal woman b] adult woman c] youn adult
woman d] young adult man e) children

12- pyloric end of stomach removed a] inc baseline acid in blood b] inc acid in nlood c] inc
stomach emptying d] inc gastrin in blood e] dec gartric compliance

13- osteomyelitis of mandible (bas itna he tha aur kuch hint nai thy) a] staph b] treponema c]
psedomonas d0 klibsella

14. ecg finding abnormal among following a] pr seg 0.1 b]lead one r wave inverse and 3 upright
c] t wave ventricle repolarization d) qrs complex ventricle depolarization

15- ketamaine is given to a burn pt because


a] causes no hypotension ufffffffff b] im and iv both c] greater analgesia uffffff

16- (is q pa cpsps ne galiyan khai hn jitni ati thein mje ) small amount of cortisol is important for
maximam physiologic affects of
a] insulin glucagon b] insulin epinephrine c) epinephrine glucagon d) androsterone epinehrine e)
thyroxine epinephrine

17- recptive relaxation of stomach decreased in


a] vagotomy b] vip c) gastrin injection d) inhibiting sympathetics

18- typhid carriers are a) aysmptomatic b) isolate and treat

19) knee joint is


a] hinge joint formed bw femur tibia and fibula b) ant cruciate ligament arises from area ant to
lunar shaped miniscus c) lat ligament is strong d) consists of two minisci d) collateral ligament
ka kuch tha

20- lower trunk of brachial plexus injury loss of function of


a) biceps b) deltoid c) opponens pollicis ( 2 aur b kuch thay )

21. cpsp favourite . Dorsal column medial leminiscus (i hate it) damage leads to loss of
a) pain touch temp pressure b) sensory ataxia C} motor ataxia two more options

88

22- left cerebral hemisphere damage near sagital area fuction loss
a] rt face b) left face c}left lower limb d) left uper limb e) rt lower limb

23- a woman brought her girl 6 yers old with complain of bed wetting even with normal
sphincteric control (me smja adh analogue aya). doctor done ivu. (me samja bifid yreter aya) and
saw ureter that was bifid ( galiyan with thinking shaid opening puchay ) and had the lower
opening in vagina with normal opening in bladder. (g.g.g.)which of the following is cause of this
a) ectopi moesonephric tissue b) division of promesonephros c) abnormal buding of
mesonephros frown emoticon

24. 20 wks prehnant woman with massive vaginal bleed and rom high grade fever and purpura
(G). fetus aborted after two days . on examination of placenta inc neutros with normal amnion
and chorion and villi absent. this condition occurred due to (G me dic k chakar me tha)
a)fibrinogen (G) b) lymphokines c) complement d) igs

25) foraman rotendum


a) lies ant to foraman ovale b) transmits facial nerve to tyoma cavity c) ant lat to pata nai kia D)
ridge is pata nai kia (G)(G)(G) option easy b ho to kuch esa dete hn k banda sai kartay huay b
pareshan ho jata ha k kahein ye new allien jaga jawab na ho

26) spleen the favourite of cpsp. old blood cells are removed from spleen in
a).......... capillaries and red pulp b) ......capillaries and spleenic cords c)......capillaries and
sinusoids d)red pulp and sinusoids e)cord and sinusoids..(deshed line me ek word tha yad nai aa
ra.

27. t4-t8 level of spinal cord injury with effects , lesion level
a)t4 b)t8 c)ant horn d) .....

28) cns tumor due to radiation


a) glio blastoma b)lymphoma c) glioblastoma multiphorme d) meningioma

29- which has no hormone secretion


a)choriostoma b) choriocarcinoma c) osteoblastoma d)anrehommoa e) teratoma

30) breast is hard leather like with pimling of the skin like onion
a) lymphatic obs b) venous obstruction c) lyphatic and venous obstruction d)invasion and
fibrosis with venous obs d) invasion and fibrosis

89

31. breast
a) mammary glands form lactation tissue of breast b} sebacious glands form on mastitis

32- loss of gaba nerons of substantia nigra and globus peluudis


a) parkinson b) hemibellismus c) chorea d) athtosis

33. pleuro,,,,, coelem and pericardio .... coelme meet in mid line to form
a) mesocoeleme b).... c

34. flexor carpi ulnaris nerve supply

35. extensor carpi ulnaris blood supply

36, wiged scapula nerve injured


37. midshaft of humrus nerve injured affects

38. ant muscles of the limb flaccid ,post normal. nerve inj

39. dorsiflexion affected nerve injured

40. eversion lost muscle ?

41. protamine side effect

42. warfarrin ovedose what givn.

43. ffps best in , warfarin

44. balochistan sibi foji cme wala senario with kalazar

90

45. cancer pleomorphism wala

46, meigns syndrome

47. dic wala ek tha

48. itp wala ek tha

49. femoral hernia a) ant and medial to pubic tubercle b)below lateral to pubic tubercle

50) japani bacha with kawasaqi disease

51. submandibular gland


a) lingual atery forms groove superiorly b) supplied by facial

52. parotd gland posterior medial relations


a. tmj post b. medial pterygoid muscle c. pharync d. carotid sheath

53. post belly of digastric nerve supply

54. ansa cervicalis forms from

55. sterocleidomastoid supply

56. inferior lip se derived from

57. labius inf with HSV lyphatic drainage

58. intapelvic nerve

59. perineum blood supply

91

60, ureter constricted at


a. psoas b. tip of t process of L veertebra d. ischial spine. d. ischial tuberosity e. sacroilliac joint

61. prostate ka kuch tha

62. infection of wound resistent to antibiotics a.staph species b.kliebsella c.pseudomonas d.ecoli.

63. fisherman with blindness

64. in drug abuser infective endocarditis wala

65. medial side of foot supply

66. ionin

67. epicardium is covered by


a. serous layer of mesothelium b. parital layer of serous mesothelim. c. visceral layer of serous
mesothelium

68, statorhea a)pancreatectomy

69) woman with megaloblasts a)pernisoius anemia

70. branch to cardiac plexus superficial a)left vagus

71. risk factore for ca. a) smoking b) asbestosis

=-=-=-=-=-

92

(7)Gyane/Obs 10th May-2016(Evening) by Dr-Haya Sayed (150)


Compiled by Nimra Ashraf

1..Bar body test in saliva is diagnostic for ans..Turnersyndrome


2..severe hypomagnesemia ans..hypocalcemia
3..test for dry eyes and dry to my (sjogren syndrome scenario) ans.Anti-SS
4..after maximum inspiration when maximally inhaled ans..vital capacity
5..MHC associated with ans..HLAgene
6..amniotic membrane ans..nonimmune genic
7..aids spread by a.oro fecal route b.blood transfusion(ans) c.droplet d.maternal fetal
route
8..potent chemotactic factor ans.C5a
9..exogenous steroid causes ans..eosinopenia
NOTE..steroids causes neutrophilia,eosinopenia and lymphopenia.and causes
decactivation of neutrophil adhesion molecules and impair migration from vasculature to
the site of inflammation..reference FIRSTAID
10..brain damage.Hemiplegia,typeofnecrosis ans..liquefactive necrosis
11.platelet adhesion ans..rough endo thelium
12..diabetic footulcer ans.angiopathy+neuropathy
13..period between S1AND S2 ans.Ventricular systole
14..bucco pharyngeal membrane ans.separates stomodium from foregut
15..factor not present in intrinsic pathway ans,factor7
16,..actin,myosin and clarthin helps in ans..pinocytosis
17..which tumor marker raised in hydatiform mole ans..beta-Hcg
18.pyogenic meningitis ans.Decreased glucose in CSF
19..doctor tell about permanent disability of child to the patient ans.denial

93

20..lesion of opticchiasm ans.bitemporal hemianopia


21..regarding pituitary what is correct ans.drain in to dural venous sinus
22..ovarynotfalldueto ans.uterus
23..female inguinal canal contains ans..round ligament+inguinal nerve
24.nerve supplyof vulva ans..pudendal nerve
25..incisionalhernia,first layer to b dissected ans.transversalis fascia
26..spinal shock ans..loss of vasomotor tone
27..herniation of disc ans..involves spinal nerve+post.ramus
28..apexofheart ans..8cm awayfrom sternum in mid clavicular line
29..bladderpartdevoidofperitoneum
NOTE..Theupperpartoftheposteriorsurfaceofthebladderiscovered
byperitoneum
The lower part of the posterior surfaceis devoid of peritoneum.
The superior surfaceofthebladderiscoveredcompletelybyperitoneum.
Along the lateral margins,theperitoneum passesontothelateralwallsof
thepelvis.This meansthatanteriorandlateralpartsofthebladderare
devoidofperitoneum.
Asthebladderfillswithurine,theperitoneum peelsofffrom theanterior
wallofpelvissothatthebladdercomesindirectcontactwiththeanterior
abdominalwall..
30..respiratorydeadspace ans.humidifiesandwarmsinspiredair
31..SER ans..detoxification
32,,breasrcarcinoma.preventionofofosteoporosisinoldage ans.raloxifen
33..aldosteroneincreasessecto ans,.hyperkalemia
34..ADHdecby ans.alcohol

35.Uwave ans..hypokalemia
36..JGcellssecrete ans..renin
37..duringexercisedecTPR dueto ans.localmetabolites

94

38..coronarybloodflowregulatedby ans..localmetabolites
39..carotidsinus ans..baroreceptor
40..oxygendecreasesinarterialbloodthanalveolarairdueto ans.pressuregradiant
41..causeofmetaplasia ans.chronicirritation
42..metaplasiadefinition
43..proteinuriadueto ans,.glomerularbasementmembranedamage
44..metabolicacidosiswithnormalaniongapin ans..diarrhea
45..proteinuria,noselesion,vasculitis ans..wegnergranulomatosis
46..biceps ans.C6
47..MMRdefinition
48..prevalencedefinition
49..cohortstudy
50.incidencedefinition
51..cervicallymphdrainage
52.righttotracheam ans.vagusnerve
53,..hyalinecartilage ans..novisiblefibres
54..generalsensationofmiddlecranialfossa ans.trigeminalnerve
55..6yearoldchildwithfinenystagmus,nootherophthalmological
andneurologicalsign,lesionin
a.occipitalcortex
b.cerebellum
c.brainstem
56..RBCsformationin2ndtrimester ans..Liver
57..Rhimmunoglobulin ans..igG
58..cataractcausedby ans.congenitalrubella

95

59..ionizing radiation causes ans.leukemia


60..farmer workingin hot environment suddenly collapsed.Na129 mEq/l ans..excessive
sweating
61..localcauseofedema ans..allergy
62..phrenicnerve accompanied by ans..pericardio phrenic artery
63..functional layer of endometrium
64..which layer shed during parturition and menses
65..not found in deep perinea lpouch ans...perinea lbody
66..lymph drainage below hymen ans..medial group of superficial inguinal lymph nodes
67..childrenwithgeneralizededemainvestigation ansurine albumin
68..fordiabeticnephropathy ans..urinary albumin
69..cocaine ans..vasoconstriction
70..surfactant deficiency ans..dec compliance
71..decreasespindlefibrefirewhen ans..muscle stretch
72..golgitendonorgan ans..muscle relaxation
73..mitochondrion ans..maternal inheritance
74..collagen ans..vitamin C
75..downsyndrome ans..robertsonian translocation
76..uremic nephropathy ans..normo chromic normocyticanemia
77..hypothyroidism druguse ans..levothyroxine
78..hasselbachtriangle ans..direct inguinal hernia
79..failureofovulationdueto ans..prolactin
80..counselling definition
81..regarding RMP NOTE..achieved by Kefflux maintained by Na/Kpump

96

82..abnormal dec in estrogen and


Inc FSH
ans..primary hypogonadism
83..0.5% bupivacaine in 100ml ans..
84..drug which require minimal absorption should have
a.low therapeutic index
b.extensive1st pass liver metabolism
85..in diabetes polyphagia due to
a.Inc glucose
b.dec sensitivity of satiety centre for glucose(ans)
86..old age diabetic type 2 drug of choice ans..metformin
87..patient having glabenclamide and also having severe asthmatic
Attack taking drug for this having blood glucose 40mg,which drug
Interact with glabenclamide
a.salisylate
b.chloramphenicol
c.b.blocker
d.steroids(ans)
88..21-23 daysmenstrual cycle,ovulation wil loccur on
a.10(ans)
b.15
c.18
d.20
89..TB diagnosis ans..AFB
90..actinomycosis ans..granuloma+abscess
91..helperTcell ans..CD4
92..helperTcells
a.decinautoimmunedisease
b.produceplasmacells
c.producecytokines(ans)
93..endarteriespresentin ans..vital organs
94..chemicalcarcinogen ans..benzidine

97

95..a man having gono coccal infection.infection spread to the Structure lying beneath
the inferior layer of urogenital diaphragm pelvic and superficial fascia
ans..superficial transverse muscle(NOTE..all others were deep content)
96..inferiorrectalarteryisbranchof ans..internal pudendal artery
97..below and lateral to pubic tubercle,2 cm below inguinal ligament ans..obturator
nerve
98..in last trimester placental membrane ans..syncitio trophoblast+fetal endothelium
99..renal column ans..inter lobar artery
NOTE..medullary rays=collecting duct
100..total ATPin glycolysis ans..38
101..left adrenal gland drain into ans.left renal vein
102..urinarysystem derivedfrom ans..intermediate mesoderm
103..when ureteric budfail to reach mesonephric cap a..kidney without ureter
b.ureterwithoutkidney
104..incTSH,decT3andT4 ans..hypothyroidism
105..positive feed back example ans..GnRH released by estrogen
106..ocps prevent ans..ovarian cancer
107..terminal ducts in breast form ans..lobule
108..HbF 97%, HbA2%..type of anemia ans..thalessemia major
109..iron deficiency anemia scenario
110..60 year old man,Hb6,TLC2.1*10^9.plateletverylow,and
Hypocellular bone marrow
a.mylofibrosis
b.acuteleukemia
111..in man after prostatectomy,fecolith in urethra
,which fascia should be preserved
ans..denonviller fascia
112..after 15 days of C-section urinary incontinence ans..vesico vaginal fistula
113..intra epithelia llesion in vulva a..paget disease b.condyloma

98

114..lactation not occur in pregnancy even prolactin level is high,reason


a.estrogen
b.progesterone
115..chronic hepatitis histology a.fibrosis b.lymphocytes(ans)
116..cloacal membrane
a..both ectoderm and endoderm at 7th week
b..formation of allantois
117..Broca area blood supply ans..middle cerebral artery
118..LH acts through ans..cAMP
119..pelvic AP diameter is more than transverse
120..sorbitol=alcohol sugar
121..telomerase action
122..testicular feminization syndrome question
123..benign tumor ans.warthintumor
124..microscopic feature of cancer ans..invasion
125..difference between benignand malignant tumor ans..metastasis
126..negative feed back by sertoli cells ans..inhibin
127..cheap method of early diagnosis of cervicalCA is ans.PAPsmear
128..fetal adrenal releases=DHEA
129..onset of parturition ans..fetal cortisol
130..amniotic embolism occurin
a.duringlabour
b.duringlabour,parturition,c-section
c.duringlabour,parturition,intrapartum
131..NOTE..3 question regarding breast lymph drainage
132..rectum posteriolylies ans..S1S2S3

99

133..alpha agonist ans..mydriasis


134..parasympatheticaction ans..miosis
135..potent stimulus of insulin release ans..GIP
136..lipid soluble local anaesthetic ans..more potent
137..bupivacain safe dose ans..150mg
138..more lipid soluble ans..more absorption
139..6 year old boy with enlarged scrotum,painandinc17- ketosteroid
a.congenital adrenal hyperplasia(ans)
b.testicular carcinoma
140..which metabolite of fatty acid metabolism enter into krebs cycle
a.acetylCo-A(ans)
b.ketones
141..incancerscreeningtestisimportant
a..most acceptable for people
b..early diagnosis(ans)
142..ant fontanelle and posterior fontanelle difference
143..black water fever=malaria
144..Quarantine malaria=plasmodium malaria

145..largestpro-erythrocytestage=plasmodium malaria

146..multiple fractures,BP90/40.initial step of management ans..I/Vfluids


147..surfactant deficiency causes inc surface tension and dec compliance
148..precursor of catecholamines ans.tyrosine

149..regarding hymen=stratified squamous epithelium

150..regarding bio availability=oral route

=-=-=-=-=-=-=-=-=-=-=-

100

(8)Gyane/Obs 11th May-2016(Evening) by Nourin Hameed (180)


1.

True about membrane transport A.glucose transport is carrier mediated B. NA moves against the conc
gradient C. K moves in a bulk flow D.water transport across membrane is always active transpose ........ 11may

2.

True about bupivacaine A. It's an ester local anesthetic B. O.5 in 50ml solution mean 50mg dose C. It
accelerated heart D. It is given in IV in regional anesthsia

3.

Global Blindness cause Ans Chlymydia

4.

Thalasemia percentage in Pakistan Ans 5%

5.

Patients is 6 cm dilated at -2 station baby s head is occipitoanterior position to take csf catheter is introduced
it wall pierces Wht structure A. Saggital sure B. Coronal suture C.anterior fontanalle D. Post fontanalle(ans)

6.

Patient pregnant.She had inverted ischial spines and narrow subpubic arch.Best term used for this type of
pelvis is
A)Contracted Pelvis
b)Anthropoid pelvi
c).Android(ans)

7.

Wht is the action of estradiol A. Glycoprotein (or may be some other thing not fat for sure ) deposition in Brest
B. Prevent bone resorption(ans) C. Make endometrial glands more tortuous

8.

Impotant landmark for assing hight of fetus during labor is Ans Ischial spine

9.

Normal HB is Ans HBA

10. During second stage of labor which substance is helpful Oxytoci(ans)


Abdominal muscle contraction
Ans

101

Oxtocin

11. Milk secretion is due to the effct of which hormoe Ans Prolactin

12. During pregnancy growth hormone is not released due the effect of which placental harmone?
Ans Human chorionic somatotropin

13. Fetal blood leaving placenta Wht percentage of Po2


A40
B 80
c. 30 .(ans)
D 60

14. Wht percent of extracellular fluid is plasma A 30 B 20 C 60 D40(answer)

15. Detruser muscle contraction is caused by


Pelvic splanchnic and hypogastric plexus

16. The endothelial molecules ICAM 1and VCAM 1 are responsible for
A.leukocyte adhesion (ans)
B.leukocyte margination

17. Which is an alpha 1 blocker Doxazocin

18. A lady had a c.section and blood transfusion 10 years back.now presented with ascites and edema. Which is
best investigation for her?

Ans
Serum electrolytes , LFTs, serum urea.

102

19. Cardiac failure mein edema kiasy hota hai us ka kia ans tha Ans Constrictive Pericarditis

20. Co-effeicient of rela +1

21. Uterosacral ligamnt waly ka? Ans Cervix to sacrum

22. Urogenital triangle nerve damage ilioinguinal? Ans

23. Counter part of mullerian tubercle An Seminal colliculus

24. Countrart oflabia majora An Scrotum


25. Associated abnormality with bicornuate uterus...kidney blader?
Ans

26. One of following is the rapid effective action of insulin


It causes k+ ions to be blocked
Inhibits gluconeogenesis and .
Movemnet of K inside cells
Ans
C

27. Immediate effect of iflamation Ans Dilation of vessel

28. Blood loss in 5 min.What happens


A)Splnchnic vasodilation
B)vasoconstriction
Ans
B

29. Effect of beta blockers


Ans

103

Vasoconstriction in muscel
30. pt has tuberculous pericarditis now presented with edema. Cause of edemaCCF. Military TB. Constrictive
perucarditis

31. Correct about ovary


Lies posterior to ureter
Attached to uterus by round ligament of UTERUS
Supplied by anterior aorta in embryonic life
Ans
C

32. urogenital sinus divided in upper and lower part by


Ans
Cloaca

33. epiploic foramen relations?


Ans
Superiorly by Liver

34. True about vagina


Posterior fornix lined by peritoneum
Supplied by vaginal and uterine arteries

Ans
B
35. Neural plate lateraly cels form?
Neural crest cells which migrate from lateral ends of hindbrain form
Rt atrium
Papillary muscle
Ans
A
36. Buccopharyngeal membrane
A)Separates Stomodeum and foregut
B)Separates orbit and nasal acvity

104

C)Separtes oral cavity from pharynx


D)Separates nasal cavity from Pharynx
Ans
A

37. True about hypophysis cerebri


Develops partly from foregut
Drains into dural venos sinuses

38. Pituitary gland true statement


Supplied by superir hypophysial arteries from internal carotid
Posterior pituitary contains acidophills and basophills
Its lesion causes unilateral hamianopia
Ans
A

39. Sup thyroid artery brach of?


Ans
External Carotid

40. Regarding the histology of the bone,which ...one is correct


A-Compact bone is consist of concentric lamellae which are continous with trabecular bone on the inner side

41. Cells with irregular foot processes present in interneuron space and around blood vessels in cns
fibrous astrocytes
protoplasmic astrocytes
Ans
B
42. Csf quantity?
5oo ml/day

105

43. True about CSF study


Ans
Low glucose in pyogenic meningitis

44. Correct association


BICEPS C6

45. Aortic valve opening outsets


Isovolumetric contraction
Isovolumetric relaxation

46. 2nd heart sound produced by


Aortic and pulmonary valve contraction

47. One was femal haver cancer on hormonal therapy mechanism


Ans
Apoptosis

48. 1 was left vocal cord paralysis ..


Ans
RLN

49. regarding heart true is


Both arteries anastomose eachother
Both arteries run between auricles and infundibulum centriole (this was exact wording)
Bot arteries have mojor branches with same names

50. Stratum functionalis is formed by


Stratum compactum and stratum spongiosum

51. One was adrenocortical insufficiency

106

52. 1 about rubella. ?


How to prevebnt rubella
Ans
Avoid contact

53. About vertebra longest transverse proces? ?


Ans
C7

54. C1 absent body

55. Lead shield used for protection in pregnant women during


Xray chest
Pelvimetry ( xray pelvimetry was not written)

56. A female with HIv have chance of which malignancy ?


Leukemia
Ovarian CA
Ans

57. A patient presents with moon face, stria on legs, truncal obesity bsl=200mg ,b.p= 150/110 (no value of acth
given) what is your diagnosis?
Crushing syndrome
Crushing disease
Ans
B
58. A pt presented with weakness, malaise, joint pain for 2 weeks. Best investigation
Blood for cold agglutinins
Blood culture
Ans
A

59. GVE Forbes of the vagus arise from


A.dorsal nucleus
B. Nucleus ambigus

107

Ans
A

60. Female presented with 104 fever right loin pain ,fever wbc=25000 .what ia the best investigation to be done ?
Urine culture
Blood culture.

61. Ek folic acid deficiency wala b to tha k fetus main kia hoga anencephaly

62. IGA % = 70%

63. Activation of cAMP mechanism is an action of A. Receptors B. Enzymes C.carrires

64. Among following Wht mostly activates complement system A.endotoxin B. Dextran

65. 2,Cerebral malria p.falcprm

66. h202,,,peroxysome

67. cardiac muscle long r.period

68. .heart most likely....heart sound anatomic area

69. True about epithelium


Simple squamous epithelium is found where exchange of gases take place

70. left shift curve...hypothermia


71. .breast atrophy adult estrogen n progestron
Ans
Estrogen only

108

72. .long transverse short ap diameter...platypoid markd

73. prolectin inhibt ovalution


74. dicroml enhance bleeding

75. .sniro lest blood...skelat muscle


76. .sniro.fat embolism
77. .scniro amniotic embolism
78. .histology bone wala

79. most likely uterine tube ..cilated epithelium


80. .True regarding breast
A)Arises from mammary bud with mammary lines
B)Doesnt arise from place other than thorax
C)Mil come inside from Montgomery tubercles after sucking
D)Size of breast determines the amount of milk in it
E)Hairs around areola ae abnormal and show hormonal influences
Ans
A

81. scnirio eating more smal feet n hand..prider wlia

82. ,In Spleen worn out blood cells come out at which part
A)Red pulp and sinuses
B)White pulp and venules
C)Penicillar capillaries and venules
D)Arteriole and penicilar capillaries
Ans
D
83. blood supply to primary cortex

109

84. A semiconscious patient aspirate while LYING on his back , the segment of the lung most
likely affected would be ;
a) anterior segment of right upper lobe
b) apical segment of rt. lower lobe
85. dopamine steady state: 9 min

86. Middle rectal artery i a branch of


Ans
Inferior vena cava

87. glucose transport through placenta via : fascilatd diffusion

88. lymphatic from vulva: sup inguinal lymph node

89. breast ca involves ant pectoral grou...upper outer quadrant


90. :2% lignocaine in 4ml :80 mg
91. : blader blood supply :ant branch of internal iliac artery

92. Example of Secondry Cartilgenous Joint


Ans
Symphysis Pubis

93. most common anemia in pragnancy: dilutional anemia


94. 53: craniopharyngioma: HMG followed by Hcg
95. amenorhae during action due to : prolactin

96. inguinal canal : sup bound by conjoint tendon

110

97. urinary bladder: stimulated by stretch receptor in the wall


98. Carotid body stimulated by
A)Increase in pH
B)Blood Pco2
Ans
B

99. bio static : mean & standard error of mean

100. Vessel damaged when incision given in uprapubic area


Ans Inferior Epigastric artery

101. common intermediate in glycolysis & glucogenesis :


A)fructose 1 6 bipiosphate to fructose 6 phosphate ans
B)Fructose 6 phophate to Glucose 6 phophate

102. In a patient occulocardiac reflex has been stimulated causing severe bradycardia and hypotension. Most
appropriate initial management:
1. IV atropine
2 removal of stimulus ans
3 hyperventilation of parient
4 anesthetize rectus muscles
5 Retrobulbar block

103. shift of curve to the left: hypothermia

104. female took tb treatment 10 yr back nw presents with generalised edema ....cause: constrictive pericarditis

105. Recurrent abscess by staph aureus n aspergillosis inf in young boy cause is: Nadph deficiency

111

106. 80: radiation injury : free redicals

107. question not rember BT ans is pie chart

108. Best source of Factor 9: ffp


109. 90: Calcitonin secreting tumor related to thyroid

110. detrusor supply: s2 s3 s4


111. chronic intra vascular hemolysis : DEC heptoglobin
112. cervical dysplasia :
multiparity / iucd?????
113. genetics : pylotic stenosis chances inc in siblings
114. obesity hirsuitism pigmentation: primary inc of ACTH
115. hormone therapy: apoptosis

116. Which one of these has has air borne transmission A. Mycobacterium B.cornybacterium

117. STD chlaymadia ya gonnorhea??


Ans
A

118. Gynae 11th may Question evening


Edema in pt of cardiac failure , result from increase formation of interestitial fluid it is due to
a) decreased colloid osmotic pressure
b) increased capillary pressure secondary to increased venous pressure
c) salt and fluid restriction by kidney
119. Patient came in ER with cardiogenic shock .. Wht first treatment A.dopamine B.adrenaline
120. Hormone causing increase glucose ,fatty acid i the blood
Ans

112

Glucagon

121. Best Ix of 20 wks gestation with previous history of hemolytic anemia in new born
Blood group B+
Ans
with B-IVE

122. Odontoid process of axis on atlas -transverse ligament

123. NahCO3 is renal compensation for


Resp alkalosis
124. Whip like movements are in
Ans
Falllopian tube

125. Monocyte
Ans
Coverts in multinucleated giant cells

126. Sensory pain sensation to uterus


Ans
Broad ligment

127. P53 gene mutation means


Ans
Cell survival

128. Causes of hepatocellular carcinoma in developing countries


Ans
Hepatitis Bad n C

113

129. High cardia output in


Ans
Beriberi

130. Terminal Ileum resection interferes with absortion of


A)Vit D
B)Calcium
C)B12
An
C

131. A case of people with smoking and healthy group.Type of study


Ans
Case Control
132. HRT Case.Mode of presentation was asked
Ans
Pie chart

133. Pregnancy cause of anemia


Ans
Dilutional anemia

134. Scenerio of Infective endocarditis


Ans
Blood culture

135. A case of haemolytic diseae of newborn.Test done inmother


An
Indirect Coomb test

114

136. A neutropenic ,hypotensive patient was admitted in oncology ward with istory of dysuria.Blood culture yielded
Pseudomonas.Substance rekeased?
Ans
Tumour necrosis factor

137. After radical prostactectomy,catheter removed after 2 weeks and patient deveolops fecaluria.Which fibrous
structure shoud be eft intact
Ans
Denoviller fascia

138. Both wwernich and Broca involvement causes


Ans
Global Aphaseia

139. H2O2 formed by


Ans
Peroxisomes

140. Type of necrosis in brain and spleen


Ans
Coagulative necrosis

141. Shivering centre is in


Ans
Posterior Hypothalamus

142. P value for


Ans
Probability

143. Median

115

Median is the one with equal upper and lower numbers

144. Mechainm of acncer


Ans
Expression of proto-oncogene

145. Doctor behaviour which attract patient


Ans
Maintain patient dignity

146. patient with multiple fractures of pelvis having loss of sensation in perianal area. which nerve is spared ?
A. ILLIOHYPOGASTRIC NERVE
b. pudendal nerve
c. ilioinguinal nerve
d. podt cutaneous nerve of thigh
Ans
A

147. Pudenadal ki root valve


Ans
S2,3,4

148. MCV=115
Ans
Macrocytic anaemia

149. Glucoe transported to placenta via


Ans
Facilitataed diffusion

150. Anticholinergic drug can be used in


A. Acute pulmonay fibrosis
B. Alergc asthma
C.chronic asthma

116

D.Acute bronchospasm
E.acute bronchitis
Ans
D

151. In interstail presure b ta


Ans
Lymphatic obstruction

152. Half life of dopamine. 9min /2


153. Short half life wala. Reaches steady state quickly / low therapeutic value
drug having short half life... a ) are extensively bound to plasma proteins.. b ) are freely filtered through
glomeruli.. c) have low therapeutic index,, d) have low volume of distribution... e ) reach steady state
concentration quickly...
Ans
E

154. Iucd ka btao i did purulent discharge qk wahan during iucd likha
Ans
Squamous Metaplasia

155. Scenerio of down syndrome.Plamar crease etc


Ans
Triomy 21

156. Menopause is due to


Ovarian unresponsivenes
Pituatry exauation due to lueting horm
Pituatry exauxtion due to fsh
Hypolthamic unresponsiveness
Ans

117

A
th

th

157. Fetal measurement done in 7 ,8 weeks by


Ans
CRL

158. person taking carbohyderate deficient diet ? which vitamin should be supplimented?
A. THIAMINE
b. riboflavin
Ans
A
159. cyclic GMP which hormone included ?
A.Insulin
B.ANP
Ans
B
160. on characterstics of TB?
a. caseous necrosis
b. epithiloid cells

161. pt in sibbi balochistan ..camping and sleeps on floor .. fever with lymphadenopathy ..history of tick bite ?
a. kalazar b. filariasis c. malaria d. Onchocerciasis

162. Lymphatic drainage of anus


A)Vertical group of superficial inguinal lymph nodes
B)Horizontal group of superficial inguinal lymph nodes
C)Deep Inuinal Lymph nodes
D)Medial group of Superficial Inguinal Lymph nodes
Ans
D

163. Labia majora lymphatic drainage


Ans
Superficial Inguinal Lymph node

118

164. regarding bupivican ?


a. block Na channel in ionised state
b. more absorption in non ionised state
Ans
A

165. longest incubation period of which malarial parasite ?


A. plasmodiam Malaria b. ovale c. vivax d. falciperem (ans)

166. pharma mcq action of anesthetic require ?


A. lipid solubility
167. two different genetic lines derived from a single zygote ?
a. genomic imprinting
b. zygote acromegaly
c. mosaicsm (ans)
d. zygote anomly

168. Protein synthesis involves


As
Mrna,protein,Tra and Ribosomes

169. Structure deep to ubmandibular gland.Passing deep to posterior beely of sternocleidomastoid


Ans
Lingual Nerve

170. Artery joing basilar artery with internal carotid artery


Ans
Posterior Communicating artery

171. Infection outside pretrachial layer goes to


A.superior mediastinum
B.posterior

119

C.anterior (ans)
D.middle

172. A pregnant lady had fracture Neck of femur in a road traffic accident.Cause of death would be?
A)Amniotic fluid embolism (ans)
B)Fat Embolism

173. Criteria for diagnosing malignant tumour


A)Pleomorphism
B)Invasion of adjacent tissues
C)Pleomorphism
D)Dysplasia
E)Metastasis (ans)

174. Caue of cerebral malaria Ans Plasmodium falciparum

175. Cause of death in gas gangrene Ans Toxemia shock

176. Calcitonin tumor marker..thyroid Ans CEA


177. Bupivicain
A act as non ionized form
B block sodium channel as aionized form(ans)
C.Sympatheyic block to urinary bladder

178. Hormone of anterior Pituitary


Ans
TSH

179. In hot sunny day,few patiets lying under sun had some sort of injury to skin and had burn.What was the
mechanism
A)Endothelial destruction (ans)
B)Mast cells destruction
180. Minimum tensile strength is achieved in how much time Ans 3 months.
=-=-=-=-

120

(9) 9th May Morning Anesthesia by by Saqiba Tahir (176)


ANAESTHESIOLOGY Paper 1 9th May 2016; Morning
1- Patient with cancer; stage T4,N1,M1. What will be seen: cachexia
2- Patient with cyanosis: deoxyhemoglobin >5%
3- Massive pulmonary embolism. Where did embolus originate from: a) long saphenous
vein b) pelvic veins c) popliteal veins d) femoral artery
4- 11 years old girl, where will be the last secondary centre of ossification present in the
humerus: a) lateral epicondyle b) medial epicondyle c) trochlea d) olecranon e)
capitulum
5- Bleeding from tracheostomy. Which vessel is responsible: a) inferior thyroid artery b)
vein connecting anterior jugular veins
6- Collagen atrophy: a) injury causes bleeding b) rheumatoid arthritis
7- Characteristic of exudates: a) decreased vascular permeability b) inflammatory cells c)
increased lipids d) specific gravity < 1.012 (no option of protein present)
8- Maximum glycogen stored in: a) liver b) skeletal muscles
9- Elastic cartilage when freshly cut appears yellow and clear because of: a) elastic fibers b)
collagen fibers c) basophils
10- True about hypertrophy: a) unlimited extent b) increased DNA content c)occurs in
endocrine glands
11- Neurotransmitter release in synaptic cleft depends on a) presynaptic calcium release b)
presynaptic action potential
12- A thirsty patient with urine osmolality of 950 and serum osmolality of 297. What is the
diagnosis: a) water deprivation b) SIADH c) Nephrogenic diabetes insipidus
13- Increased vagal stimulation causes: a) increased PR interval
14- Ameboid locomotion is seen in: a) WBCs
15- Potassium secretion: a) Aldosterone b) ANP c) Angiotensin
16- Which one amongst the following is benign: a) Melanoma b) Lymphoma c) Adenoma d)
Hepatoma
17- Axillary sheath is formed by: a) pretracheal fascia b) prevertebral fascia c) superficial
cervical fascia
18- ICAM and VCAM: a) leucocyte adhesion
19- Hyaline cartilage present in: a) Intervertebral disc b) Pinna c) TMJ articular d) Epiglottis
20- Secondary cartilaginous joint is: a) Pubic symphysis
21- Anterior relationship of right kidney: a) Liver b) 2nd part of duodenum c) hepatic flexure
d) small intestine

121

22- In normal quiet respiration, resonance of chest in mid-scapular line posteriorly is


present till: a) 7th rib b) 8th rib c) 9th rib d) 10th rib e)11th rib
23- G6PD deficiency presents most commonly as: a) drug induced hemolysis b) favism
24- A girl was asked a question in a lecture; her face became red. What is the mechanism:
a) Active hyperemia b) Passive hyperemia c) Increased congestion
25- A man with history of chronic alcohol abuse but no medical illness is working well and
comes to a doctor for routine checkup. Examination is unremarkable. AST=40, ALT=40,
Total bilirubin =1.1; most likely microscopic feature is: a) fatty liver b) hypertrophy of
smooth endoplasmic reticulum
26- Parasympathetic stimulation causes: a) relaxation of detrusor muscle b) contraction of
sphincters c) erection d) decreased gut motility
27- Abnormal protein synthesis. Misfolded proteins degraded by: a) Peroxisome b)
Proteasome c) Nucleosome
28- Increased WBC count in viral infection. Which cells are responsible: a) Lymphocytes b)
Neutrophils c) Eosinophils d) Basophils
29- AIDs characterized by: a) decrease in T helper cells
30- Spinal cord is suspended in dura meter by: a) denticulate ligament b) alar ligament
31- Rapid filling of ventricle produces vibration responsible for which heart sound: a) 1 st
heart sound b) 2nd heart sound c) 3rd heart sound d) 4th heart sound
32- Test for typhoid in 1st week of infection: a) blood culture b) complement fixation c)
Widals test
33- Most potent anti-oxidant: a) vitamin A b) vitamin C c) vitamin E d) Transferrin
(glutathione not in options)
34- Aldosterone causes: a) increased tubular reabsorption of sodium b) most active
mineralocorticoid
35- Many WBCs lining the endothelium is called: a) Adhesion b) margination c)
pavementation d) diapedisis e) chemotaxis
36- Visceral afferent
37- Most appropriate about cardiac muscle is a) supplied by autonomic nervous system b)
striated
38- Osmolarity of normal saline in mOsm/L : a) 280 b) 320
39- Research about risk of MI and level of exertion 24 hours prior to MI. 3 result values
given for mild, moderate and severe level of exertion. This is best represented by: a) bar
chart b) pie chart
40- 4 fold or 2 x 2 contingency table: Chi square test
41- Regarding mode of action of aspirin: a) COX inhibitor
42- Vitamin D deficiency causes: a) Rickets
43- Iron absorption increased by: a) Ferrous form b) oxalate intake

122

44- Cortisol causes: a) decreased intake of glucose into cells


45- Girl talking to a doctor in a seductive tone. What should doctor do: a) refer to other
doctor b) ask open ended questions c) call in a nurse
46- Factor 8 released by: a) endothelium b) liver
47- 4th layer of scalp is: a) loose connective tissue b) dense connective tissue
48- Which drug acts by inhibition of H+/K+ pump: a) Omeprazole b) cimetidine c) ranitidine
49- Infection between investing layer of deep fascia and pretracheal fascia. Where can it
spread: a) retropharyngeal space b) posterior to sternocleidomastoid c) superior border
of manubrium sternum d) anterior to pericardium e) oesophagus
50- Right bronchial artery arises from: a) 2nd intercostals artery b) 3rd intercostals artery c)
thoracic aorta d) arch of aorta
51- Isthmus of thyroid is present in front of: a) 2nd, 3rd, and 4th tracheal rings
52- Secretion of proteins from kidney is prevented by: a) podocyte foot processes b)
basement membrane c) fenestrations
53- Renal clearance of a substance depends on: a) increase in GFR b) aldosterone
54- Patient with adrenalectomy. Taste preference for: a) sodium chloride
55- Sign of reversible injury: a) cellular blebs b) swelling of endoplasmic reticulum
56- Extraembryonic coelom derived from: a) intraembryonic coelom b) lateral mesoderm c)
paraxial mesoderm d) intermediate mesoderm
57- Stratified cuboidal epithelium present in: a) lining of salivary duct b) urethra
58- Structure passing through the oesophageal opening of thoracic diaphragm: a) right
vagus nerve b) right phrenic nerve c) hemiazygous vein
59- Nitroglycerine should not be given orally because: a )inactivated during first passage
through liver
60- Best investigation for diagnosing diabetic nephropathy: a) urine albumin
61- Edema in renal pathology is the result of: a) sodium retention b) hypoalbuminemia c)
sodium retention and hypoalbuminemia
62- What decreases in pregnancy: a) PaCO2 b) minute ventilation
63- Food which causes a decrease in risk of cancer: a) chocolate bonbon b) smoked meat c)
apples d) rum
64- Treatment of enteric fever: a) ciprofloxacin
65- Mothers blood group is A. Childs blood group is O. Other childs blood group is AB.
What is the blood group of the father? : a) B b) children have 2 different fathers
66- Amniocentesis shows karyotype XXY. Diagnosis: a) Klinefelters b) Turners c) Androgen
insensitivity syndrome
67- Active tubular reabsorption of: a) Streptomycin b) Penicillin
68- Medical ethics: a) covered by Hippocratic oath b) code of professional conduct c) code
of moral conduct of a doctor

123

69- Increased age causes a decrease in: a) vital capacity b) heart rate c) blood pressure d)
pulse pressure
70- Warfarin therapy in DVT: a) not to be given in pregnancy b) can be quickly reversed by
FFPs c) vitamin K will work in short term
71- Air flows to the alveoli more at the base because of: a) increased compliance at the base
b) increased blood flow at the base
72- What passes through vena caval opening of thoracic diaphragm: a) right phrenic nerve
73- Drug causing red man syndrome: vancomycin
74- Patient with stroke. Repair of brain tissue by: a) glial cells b) oligodendrocytes
75- Adenoma derived from: a) epithelial cells b) glands
76- Major buffer of blood: a) hemoglobin b) bicarbonate c) proteins

Anaesthesiology
Paper 2
9th May 2016; Morning

1- Treatment of Gille de touretter syndrome: a) Haloperidol b) Methylphenidate


2- Mapelson A breathing system. FGF 1.5 x minute volume to prevent rebreathing of
alveolar air????
3- Regarding beta selectivity: a) Timolol- beta 1 selective b) Betaxolol- beta 1 selective c)
lebuvolol- beta 1 selective d) betaxolol- beta 2 selective e) levobulol- beta 2 selective
4- True about internal capsule: a) somatosensory fibers in anterior limb b) thalamus lies
medial to posterior limb c) corticospinal fibers are present in anterior limb
5- 80 years old male unable to sleep at night. Came to a doctor. BP was 160/80. Not on any
antihypertensives. Didnt have any blood pressure problems before. Most likely cause of
increased blood pressure in this patient is: a)decreased aortic compliance b) unable to
sleep at night
6- Neuromuscular blocker with maximum release of histamine: a) Tubocurare b)
Atracurium c) Succinylcholine
7- Percentage of oxygen in a mixture of 2% halothane and air is: a) 20.4% b) 40% c) 50% d)
15.4% e)60%
8- Decreased hypothalamic function will result in an increase in one of the following: a)
prolactin b) TSH
9- L5 slides over sacrum anteriorly such that it is nearer to the sacral promontory. What is
this condition called? : a) Spondylosis b) Spondylolisthesis

124

10- Carbondioxide in tissues is transported to the alveoli mainly in the form of: a) plasma
bicarbonate b) carbonic hemoglobin
11- HR = 75 and ventricular systole is 0.3 seconds. When HR increases to 225, what will be
the duration of ventricular systole: a) 0.9 sec b) 0.1 sec c) 0.6 sec
12- TMJ dislocates mostly in which direction: a) anteriorly b) posteriorly c) laterally d)
medially
13- How does thiazide diuretics decrease edema: a) decreasing sodium absorption
14- Gyri longus and brevis present in which area of brain: a) Frontal lobe b) Parietal lobe c)
Temporal lobe d) Occipital lobe e) Insula
15- Salbutamol causes: a) bradycardia b) hypokalemia c) increase uterine contractions
16- A girl with pheochromocytoma to be operated. For control of blood pressure what
should be given: a) alpha and beta blocker b) beta blocker c) alpha blocker
17- Dobutamine primarily acts on: a) beta 1 receptor b) beta 1 and 2 receptors c) alpha and
beta receptors d) beta 1 and 2 receptors and dopamine receptor
18- In a patient with heart failure, mannitol should be given with caution because of risk of:
a) precipitating pulmonary edema b) increasing intracranial pressure c) decreasing
concentrating ability of kidneys
19- Furosemide useful in: a) acute pulmonary edema b) open angle glaucoma
20- Lady can understand speech but unable to speak. Lesion is in which area of brain: a)
Brocas area b) Wernickes area
21- Bleeding from duodenal ulcer. Most likely: a) posterior duodenal ulcer and erosion of
gastroduodenal artery b) anteriorly duodenal ulcer and erosion of gastroduodenal
artery
22- Mode of action of methylxanthines (theophylline): a) Phosphodiesterase inhibiton b)
aldosterone receptor antagonism
23- Damage to medullary cuneatus and gracilis causes: a) analgesia b) anesthesia
c)asteriognosis
24- Drug safe in pregnancy: a) Ibuprofen B) Naproxen c) ketorolac
25- Regarding electrical isolation of circuit with use of ECG monitoring for placing a CVP line:
a) use of transducer for converting electrical signals to light b) a current leak of 200
milliamperes can cause ventricular fibrillation c) using something that gives a warning
when leakage of current is of 50 milliamperes d) using a non conducting wire over a
conducting wire
26- Organophosphorus poisoning: a) Physostigmine b) Neostigmine c) Pralidoxime d)
Edrophonium
27- Cromolyn sodium: a) used in acute asthma b) all asthma c) prevents antigen associated
mast cell degranulation d) used in children cause added effects?
28- Fibers of corpus collosum most likely called: a) commissural fibers b) projection fibers

125

29- Trismus is one of the first sign of a developing malignant hyperthermia with use of IV
succinylcholine in what percentage of patients: a) <50% b) 50% c) 75% d) 80% e) >85%
30- Patient on multi-drug resistant ATT. After a while, red-orange color blindness due to: a)
Ethambutol b) Isoniazid c) Streptomycin d) Pyrazinamide
31- Key components of a sensory modality: a) type of receptor, receptive filed, adaptation
b) type of receptor, location in body, adaptation c) cant recall other options
32- Upper lung lobe has: a) increased dead space
33- Patient with blood loss between 15-20%. What should be given via infusion: a) normal
saline b) ringers lactate c) colloids d) whole blood
34- Thirst increased by a decrease in: a) baroreceptor firing b) increased plasma volume
35- Conduction velocity of propagation in an axon is increased by: a) decrease in membrane
resistance b) decrease in axon diameter c) increased refractory period d) excitability
36- Toxic effects of local anesthetics: a) bupivacaine cardiac effects seen more than that of
lidocaine
37- Solution used for scrubbing: a) chlorhexidine b) potassium permagnate c) alcohol
38- Cleaning or disinfection of skin: a) chlorhexidine
39- Most important for preventing transmission of infection in ICU: a) frequent hand
washing between patients b) wearing gowns c) wearing gloves
40- ETT breathing system ( cant recall at all)
41- Regarding autoregulation of blood from most sensitive to least sensitive: a) cerebrum >
cerebellum > spinal cord b) cerebellum > cerebrum > spinal cord
42- Regarding epinephrine: a) causes bronchodilation by acting on beta 2 receptors
43- True regarding alpha motor neurons: a) myelinated b) unmyelinated c) innervates motor
end plates d) innervates intrafusal fibers
44- PR interval best represented on JVP curve by: a) a to c wave b) a, c and x c) c wave by
atrial systole
45- Which analgesic is safest for use with MAO inhibitors: a) Fentanyl b) Buprenorphine
46- Blood supply of cingulated gyrus: a) anterior choroidal artery b) posterior choroidal
artery c) middle cerebral d) anterior cerebral
47- Lateral curvature loss causes: a) kyphosis b) lordosis c) scoliosis
48- Partial pressure of oxygen in alveoli at height of 14000 feet? Where barometric pressure
is 450 mm Hg: a) 84 b) 40 c)104
49- If pH=7.4, ratio of [NaHCO3] to [H2CO3] is: a) 1:20 b) 1:25 c) 1:30 c) 1:5
50- Greatest potential difference in the wall of stomach achieved by: a) aspirin
51- Autonomic nervous system supply of stomach: a) parasympathetic increases motility b)
parasympathetic increase secretion of enzymes
52- Re-entry circuit causes: a) atrial fibrillation b) atrial flutter c) paroxysmal atrial
tachycardia d) paroxysmal nodal tachycardia

126

53- Selective COX-2 inhibitor: a) Meloxicam


54- Digitalis indicated in: a) atrial flutter b) ventricular tachycardia
55- Greatest chance of pneumothorax while inserting CVP is with which route: a) internal
jugular b) external jugular c) right subclavian d) left subclavian
56- Regarding thermoreceptors, best is: a) large receptive field b) changing temperature
causes a dynamic response
57- Patient with RTA has memory loss. Damage to which lobe has occurred: a) temporal
lobe b) parietal lobe
58- Rapid treatment for SVT: a) adenosine b) amiodarone c) propanolol d) verapamil e)
lidocaine
59- An anti-anginal drug that causes constipation and edema: a) diltiazem b) hydralazine c)
nitroglycerine d) propanolol e) isosorbide dinitrate
60- Cimetidine causes: a) liver enzyme inhibition b) SLE
61- What ascends from thorax to root of neck, behind sternoclavicular junction: a) SVC b)
arch of aorta c) right subclavian d) left subclavian
62- 3rd lower molar tooth extraction under GA. Loss of sensation of lateral tongue on that
side. Nerve damaged is: a) Inferior alveolar nerve b) lingual nerve
63- To differentiate between myasthenia crisis and cholinergic crisis, what is used? A)
edrophonium b) neostigmine c) pyridostigmine d) physostigmine
64- Blood supply of spinal cord: a) vertebral artery b) basilar artery c) PCOM
65- Membrane resting potential is due to a) K+ efflux b) Na+/ K+ pump
66- Fetal blood leaving the placenta carry how much PO2: a) 10 b) 20 c) 30 d) 40 e) 50
67- Consistent finding of hypokalemia is present in: a) persistent vomiting b) respiratory
alkalosis c) DKA
68- Anterior surface of heart is mainly formed by: a) RA b) RV c) LV d) LA
69- 3rd space fluid loss. Replace it with: a) ringers lactate b) normal saline c) 5% dextrose
70- Fluid not isotonic with plasma: a) normal saline b) 5% dextrose c) 1.2% sodium
bicarbonate d) Hartmans solution (ringers lactate)
71- Sign of lignocaine overdose: a) prolonged convulsions b) perioral anesthesia
72- Paracetamol overdose. What should be given: a) acetylcysteine
73- Abrasion causing bleeding. 1st change seen will be: a) vasoconstriction
74- AV node supplied mostly by: a) RCA b) LCA
75- Force of heart contraction depends on: a) pre-existing length of muscle fiber b) gap
junctions
76- Increase in ESR with: a) increased fibrinogen b) albumin c) cholesterol
77- 3rd degree burn complication: a) contracture b) neuroma
78- Anion gap between :a) measured anions and cations
79- Primordial germ cells incorporated in gonads in sex cord in: a) 3rd week b) 4th week

127

80- Gyri longus and brevis are present in which area of brain: a) frontal lobe b) parietal lobe
c) temporal lobe d) occipital lobe e) insula
81- Cingulate gyrus lies in which lobe: a) temporal b) parietal c) insular d) frontal
82- Satiety is mediated by: a) impulses from GIT b) increased blood glucose levels c) uptake
of glucose into cells via insulin
83- To check vitamin D status in body, what should be measured? : a) 1 alpha hydroxylase b)
cholecalciferol c) 1, 25 vitamin D d) 25 vitamin D
84- If membrane potential decreases, it is due to: a) K+ efflux b) Na+ efflux c) Cl- efflux
85- Neural crest cell derivative: a) dorsal root ganglion b) preganglionic autonomic fiber c)
adrenal cortex d) astrocytes and oligodendrocytes
86- Partial pressure of halothane at 20 degree Celsius is : a) 184 b) 568 c) 243
87- True about propofol: a) conjugated to glucoronide and sulfate in liver b) prepared in
propylene glycol
88- Ideal circuit for controlled mechanical ventilation: a) Mapelson A b) Mapelson B c)
Mapelson C d) Mapelson D e) Mapelson E
89- Carbondioxide will be best absorbed by one of the following: a) alkalies, silicates and 1418% moisture b) barium hydroxide and calcium hydroxide
90- Tract from the cerebrum ending on anterior horn cells in lateral and central part
(controlling distal musculature) of spinal cord is a) rubrospinal b) reticulospinal (No
option of corticospinal was present)
91- Drug contraindicated in intracranial hypertension: a) Ketamine b) Propofol
92- In treatment of nausea and vomiting: a) metoclopramide is less effective against
cytotoxic drug induced vomiting
93- Feature of a non-depolarising neuromuscular blocker is: a) poor sustained tetanic
contraction
94- Cimetidine is given preoperatively to: a) reduce gastric volume b) reduce gastric acidity
95- Patient is given spinal anesthesia and became hypotensive. Cause is: a) loss of
vasomotor tone
96- Patient allergic to bupivacaine and procaine. Which local anesthetic can be used? : a)
etidocaine b) lidocaine
97- Shortest acting non-depolarizing neuromuscular blocker: a) rocuronium b) mivacurium
98- Recent memory and alertness associated with: a) anterior thalamic nucleus b) reticular
formation
99- Critical temperature is defined as: a) temperature to which a gas should be cooled to be
liquefied by pressure alone
100About pancuronium: a) steroid b) natural steroid c) synthetic steroid

Compiled by Dr. Saqiba Tahir

128

(10) Radiology 9th May (Evening Session) by Bint-e-Adam (162MCQS)


1. During a surgery a surgeon nick hepatoduodenal ligament. Which structure on right side is likely to be
damaged?
A. Portal vein
B. Bile duct (answer)
C. IVC
D. Hepatic Duct

2. Gall Bladder
A. Produces bile
B. Submucosa is present
C. Mucosa is thrown in extensive circular folds (answer)

3. Bare area of liver is limited by


A. Right and left triangular ligament
B. Falciform ligament
C. Ligament teres
D. Ligament venosum
E. Coronary ligament (answer)

4. Medial side of arm is supplied by


A. Medial cord (answer)
B. Lateral cord
C. Musculocutaneous nerve
D. Median nerve

5. Pendular knee jerk

129

A. UMN lesion
B. LMN lesion
C. Hypothyroidism
D. Parkinson
E. Cerebellar lesion (answer)

6. Intercostal spaces
A. Have three layers of muscles in between costal cartilages
B. Supplied by only sensory nerves
C. Blood supply from int. Thoracic artery n thoracic aorta (answer)
D. Neurovascular bundle is present at upper border of rib

7. Neurovascular bundle
A. Lies deep in ribs
B. Lied on upper border of ribs
C. Lies beneath lower border of ribs (answer)

8. Mesonephric duct functional remnant or unit is


A. Epididymis
B. Ductus deferens (answer)
C. Vagina
D. Uterus

10. Metastatic Calcification of kidney most commonly due to


A. Hypervitaminosis D
B. Hyperparathyroidism (answer)
C. Hypercalcemia

130

11. Main supply of head of humerus


A. Ant circumflex artery
B. Post circumflex artery
C. Arteries around rotator cuff
D. Arcuate artery (answer)
E. Subscapular artery

12. Deep cervical artery is a branch of


A. Costocervical trunk (answer)
B. Subscapular artery
C. Aorta 1st part
D. Int thoracic artery

13. Proximal part of CBD is supplied by


A. Inf mesenteric artery
B. Sup mesenteric artery
C. Cystic artery (answer)
D. Right hepatic art
E. Left hepatic artery

14. A pt. got spleen rupture and she is going to spleenectomy. Whats her peripheral blood picture right now
A. Howel jowel bodies (answer)
B. Thrombocytopenia

15. Infants spinal cord ends at


A. Btw L1 and L2

131

B. Lower border of L3 (answer)


C. L2

16. Esophagogastric junction is at


A. T11 (answer)
B. T7
C. L1
D. L3

17. Portal HTN


A. Left colic (answer)
B. Inf mesenteric
C. Inf epigastric

18. A young 1yr old has hydrocele


A. Scrotal swelling
B. Peritoneal fluid accumulation (answer)
C. Testis inflammation

19. Sigmoid and descending colon drain into


A. Left colic nodes
B. Inf mesenteric nodes (answer)
C. Sup mesenteric nodes

20. Hip joint is


A. Related post to psoas bursa
B. Has Obturator ext. inferiorly (answer)

132

C. Related to femoral nerve

21. Femur bone


A. Lateral ligament attached to lower epicondyle (answer)
B. Plantaris muscle

22. Rectum is
A. Straight structure
B. Has haustrations circular folds
C. Post to rectum is S3S4S5

23. Deep inguinal ring & Inf epigastric artery


A. Medial
B. Lateral (answer)

24. AV bundle
A. Only neuro connection btw right atria &ventricle (answer)
B. Present in right atria

25. Post interventricular artery supplies


A. Both right and left ventricle (answer)
B. SA node
C. Rt atria

26. Most medial nucleus of cerebellum


A. Fastgial (answer)
B. Dentate

133

27. Defect in interventricular septum causes or affects which valve most


A. Aortic
B. Pulmonary (answer)
C. Tricuspid
D. Mitral
E. Coronary sinus

28. Lateral side of breast drains into


A. Pectoral LN

29. Esophagus constricted at


A. Where crossed by arch of aorta (answer)
B. Post mediastinum + left ventricle
C. Rt bronchus

30. Site of radial artery pulse is


A. Ext pollicis and abd pollicis (answer)
B. Brachioradialis and flexor carpi ulnaris

31. Most imp anatomic largest subdivision of prostate is


A. Median lobe
B. Peripheral zone (answer)
C. Transitional zone

32. Pain of vagina mediated medially due to


A. Obturator nerve (answer)

134

B. Femoral nerve
C. Sciatic nerve

33. Female pelvis with short AP diameter and long transverse diameter is
A. Plateploid (answer)
B. Gynaecoid
C. Anthropoid

34. Intrinsic factor is secreted from


A. Gastric fundus (answer)
B. Gastric antrum
C. Lesser curvature

35. Sickle cell disease is a defect in


A. Beta globulin (answer)
B. Albumin

36. Lateral popliteal nerve damage scenario in a footballer and it was asked where it is damaged
A. Neck of fibular (answer)
B. Head of fibular
C. Ant to tibia

37. No of divisions of lower airways?


A. 17
B. 21
C. 23(answer)
D. 26

135

38. Reflux of cecum is prevented by


A. Ileocecal valve
B. Ileocecal sphincter (answer)

39. Right testis cancer scenario and asked about its drainage
A. Para-aortic lymph node (answer)
B. Int iliac lymph nodes

40. Prostate some scenario was given drains in


A. Int iliac nodes (answer)
B. Para aortic nodes

41. Left supra renal gland


A. Separated from left kidney by perirenal fascia
B. Drains in left renal vein (answer)

42. Parasympathetic supply to submandibular gland is from


A. Sup salivary nucleus (answer)
B. Inf salivary nucleus
C. Auriculotemporal nerve
D. Lesser petrosal
E. Greater petrosal

43. Post belly of digastric is supplied by


A. Facial nerve (answer)
B. Accessory nerve

136

44. Vertebrae
A. Post and Inf spine (answer)
B. Nerve leave sup
C. Cervical part

45. Angina is mainly present in


A. Abdominal aorta
B. Sup mesenteric artery (answer)

46. Post curvature is lost in an infant. This condition is called


A. Kyphosis (answer)
B. Scoliosis

47. A girl has problem in opening mouth. Which muscle is defected?


A. Lateral pterygoid (answer)
B. Medial pterygoid
C. Temporalis
D. Buccinators

48. Annulus fibrosis


A. Attached med and lat ligaments (answer)
B. No sensory supply

49. Phenylalanine converts into


A. Tyrosine (answer)
B. Tyrmaine

137

50. Stab wound injury in intercostal space. Surgeon needs to go to pleura by reaching
A. Ext intercostal and internal intercostal muscles
B. Neurovascular bundle
C. Parietal pleura
D. Endothoracic fascia (ans)

51. Area of visual field supplied by


A. PCA (answer)
B. MCA

52. Homonymous hemianopia caused by


A. Optic tract (answer)
B. Optic chiasma

53. Intrauterine midtrimester hematopoiesis occurs in


A. Liver (answer)
B. Spleen

54. Pancreas related anteriorly


A. Lesser sac (answer)
B. Splenic vein
C. Root of mesentery

55. Lower limit of rectus sheath is


A. Arcuate line, (answer)
B. Umbilicus or

138

C. Linea semilunaris

55. Some scenario on loss of sensation in little finger and loss of abduction of finger
A. Damage to ulnar nerve (answer)

56. Sup laryngeal artery accompanied by


A. External laryngeal nerve
B. Internal laryngeal nerve (answer)

57. Osteoporosis
A. Reduced bone density (answer)
B. Reduced bone mineral

58. Annular pancreas due to


A. Ventral mesentery (answer)
B. Dorsal mesentery

59. Ureter related posteriorly to


A. Gonadal vessels (answer)
B. Uterine artery
C. Common iliac

60. Gastric emptying is delayed due to


A. Gastrin
B. CCK (answer)

61. Oxytocin is secreted from

139

A. Hypothalamus (answer)
B. Neurophysis

62. Renal artery divisions sequence


A. Segmental, lobar, Interlobar, arcuate (answer)
B. Interlobar, arcuate, lobar, segmental
C. Segmental, Interlobar, lobar, arcuate

63.Which loops around arch of aorta


A. Left recurrent laryngeal nerve (answer)
B. Azygous vein
C. Left vagus nerve
D. Subclavian
E. Phrenic nerve

64.Foetal period starts after which week:


A. 11th
B. 8th (answer)
th

C. 12

th

D.16

st

E. 21

65.Hypothyroid patient in on thyroxin, best marker to monitor his thyroid status is:
A.T3
B.T4
C.TSH (answer)
D.T3 and T4

140

E.T3, T4 and TSH levels

66. IF THERE IS A TUMOR IN THE RIGHT SUPERIOR LOBAR BRONCHUS THE BRONCHOPULMONARY SEGMENT MOST
LIKELY TO BE COLLAPSED WOULD BE:
A.APICAL SUPERIOR (ans)
B.ANTERIOR BASAL
C.MEDIAL BASAL
D.MIDDLE LATERAL
E.SUPERIOR BASAL

67.charctristic of TB granuloma
A. Necrosis
B. Epitheloid cell (ans)
C. Giant cell

68.diff in plasma and Interstitial fluid osmolarity


A.1mosm (ans)
B.1.5mosm
C.5mosm

69.Body response in cold to regulate temp


A. Anorexia
B. Hunger (ans)
C. Vasodilation

70.Highest con of ACTH in stress seen in


A. Systemic Arterial blood

141

B. Hypophysial arteries
C. Venous blood draining Ant pituitary
D. Venous blood draining Posterior pituitary
E. Venous blood draining adrenal medulla

71. SCM is supplied by


A. Spinal accessory Nerve (ans)
B. C3, C4
C. Suprascapular
D. Dorsal scapular

72. Dec in effective circulation volume will be .


A. Inc urine flow rate
B. Inc GFR
C. INC renin
D. Inc ANP
E. INC FREE solute water excretion

73. Angiotensin is response for


A. Systemic contraction of venules
B. Glomerular afferent arterial vasoconstriction
C. Renin release
D. Stimulation of thirst center

74. 60 year old lady bluish spot on thigh BT 1 and clotting time 3 Diagnosis
A. Platelets count
B. Prothrombin time

142

75. Lymphocytes increased in


A. Polycythemia Vera
B. Cushing/adrenocortical excess
C. TB

76. Phlebo thrombus


a. Heart
b. Small arteries
c. Veins (ans)
d. Adult
e. Children

77. Most common immunodeficiency


A. Isolated igA deficiency
B. X-Linked agammaglobinemia

78. Space between splanchnic n somatic extracelomic mesoderm


A. Extra embryonic coelom (ans)
B. Amniotic cavity
C. Blastocoele

79. A person transplanted a part of liver to other person having hepatic failure After some time liver size increased
in both due to
A. Dysplasia
B. Metaplasia
C. Hyperplasia (ans)

143

D. Tumor
E. Hypertrophy

80. Brocas area and its near by area supplied by?


A. Ant cerebral
B. Mid cerebral (ans)
C. Ant choroidal

81. Lateral to carotid artery


A. Jugular vein (ans)
B. Sternocleidomastoid
C. Clavicle
D. Trachea
E. Manubrium

82. Protinemia adverse effect


a. Hypotension (ans)
b. Pulmonary vasoconstriction
c. Bleeding
d. Anaphylaxis

83. Neutrophil collected from blood are analyzed for burst of oxygen consumption from which of the following
even in acute inflammatory response to respiratory burst is essential
a. Attachment of endothelial cell
b. Increase production in bone marrow
c. Oppsoniazation of bacteria
d. Phagocytosis of bacteria

144

e. Microbial killing

84. Most frequent etiological hepatitis in our country


a. Hep a
b. Hep b (ans)
c. Hep c
d. Hep d

85. Human epicardium consist of


a. Visceral and parietal layer
b. Myocardial cell
c. Visceral serous pericardium
d. External connective tissue

86. Diarrhea
a. Metabolic acidosis with normal anion gap (ans)
b. Metabolic alkalosis with normal anion gap
c. Metabolic acidosis with abnormal anion gap
d. Metabolic alkalosis with abnormal anion gap

87. ECF connected to cytoskeleton


a. Proteoglycan (ans)
b. Integrin
c. Cadherin
d. Intermediate filament

88. Newborn with recurrent infection which physiological antibody is missing

145

a. igG
b. igM
c. igA
d. IgE

89. Patient is taking glucocorticoid have Dec in


a. Lymphocyte (ans)
b. Neutrophil
c. Monocyte

90. 8% blood loss in 30 min with no change in atrial pressure which of the following has maximum lost of blood
a. Aorta
b. Arteries
c. Arteriole
d. Capillary
d. Veins

91. Sympathetic action on alpha-adrenergic receptor


a. Increase heart rate
b. Contraction of ciliary muscle (ans)
c. Bronchial dilation

92. Sick disease


a. Patient no taking medicine

93. Autosomal dominant


a. Marfan (ans)

146

b. Phenylketoneuria
c. Gaucher
d. Alkaptonuria

94. Tumour without hormone production is


a. Arrhenoblastoma
b. Chondrsarcoma
c. Carcinoid
d. Teratoma
e. Choriocarcinoma

95. Patient with EcG pR interval 40


a. complete heart block,
b. Mobitz type1
c. Mobitz type 2

96. Cardiac plexus in superior mediastinum receives input from


a. Right vagus
b. Left vagus
c. Right cervical sympathetic
d. Internal laryngeal

97. Facial nerve supply


A. Tensor tympani
B. Posterior belly of digastric
C. Posterior auricularis
D. Pure motor
E. Chorda tympani arise from horizontal plate

147

98. High cardiac output failure


A. Niacin
B. Riboflavin
C. Thiamine

99. End effect of sepsis


A. DIC

100. Mitosis and pairing of chromosomes occur in


A. Anaphase
B. Metaphase
C. Prophase
D. Telophase
E. Early Telophase

101. Fluid in distal proximal convoluted tubule is


A. Low PH when acidic urine excretes
B. Increase urea than bowman
C. 15% absorption is under effect of ADH
D. Osmolarity is half of GFR
E. Glucose concentration is same to that of plasma

102. Sympathetic through alpha-adrenergic


A. Control radial muscle of iris
B. Melatonin synthesis
C. Bronchial relaxation

148

D. Increase Heart rate


E. Glycogenolysis

103. In 60 year male left hemicolectomy done due to DUKE B tumor and there is no liver Mets best prognostic
level, which should b measured
A. B hcG
B. Bilirubin
C. CEA
D. AFP
E. Alkaline phosphatase

104. Coagulative necrosis


A. By Ischemia
B. By viral infection
C. Pink homogenous area under light microscope
D. Not in kidney
E. Infancy disease

105. 1/3RD of total body water


A. ECF
B. ICF
C. Interstitial
D. Plasma
E. Lymph

106. Muscle, which depress mouth corner, e.g. sad face


A. Platysma

149

B. Orbicularis oculi
C. Buccinator
D. Mentalis

107. Mechanism of tumor metastasis


A. Migration of tumor cells
B. Attachment to matrix
C. Degradation of E-cadherin

108. Quite inspiration


A. Diaphragm
B. External intercostal
C. Pectoralis minor
D. Internal intercostal
E. Scalene

109. Increase WBCS in viral infection


A. Lymphocytes
B. Plasma cells
C. Eosinophils
D. Basophils

110.Glucose Filtered BY glomerulus and reabsorbed from


A. PCT
B. DCT
C. Loop of henle
D. Collecting ducts

150

111. 25 year old female, doing heavy exercise more body water loss occur by
A. Sweating
B. Lungs
C. Insensible loss

112. Example of hyperplasia+hypertrophy


A. Uterus in pregnancy
B. Breast in puberty
C. Skeletal muscle growth in athlete

113. True hermaphrodite


A. XO
B. XXY
C. XY
D. XX/XY

114. Increase lymphocytes in-patient of


A. Hay fever
B. TB
C. Septicemia
D. Polycythemia
E. Pneumonia

115. C1 vertebra characteristic


A. Absent foramen transversum
B. Odontoid process

151

C. Absent body
D. Long spinous process
E. Massive body

116. Smooth Muscles has


A. Gap junctions
B. Motor neuron
C. Sarcomere
D. T-Tubule
E. Motor end plate

117. Rough Endoplasmic reticulum


A. Detoxification
B. Cholesterol Synthesis
C. Protein synthesis
D. Calcium storage
E. Phospholipids synthesis

118. Iron transport is carried out by


A. Globulin
B. Transferrin
C. Albumin
D. Primary Albumin
E. Ferritin

119. Autonomic nerve function stopped by Atropine


A. Sweat gland stimulation

152

B. Pupil dilatation
C. Dilation of bronchioles
D. Excitement of cardiac muscle contractility
E. Inhibit git motility

120. Plasma membrane thickness


A. 7-11nm
B. 4-6 nm
C. 12-14nm
D. 14-20nm

121. Baroreceptor is sensitive to


A. Hypotension
B. Hypertension
C. Fluid intake
D. Decrease arterial pulse

122. CSF
A. Flow from 3rd ventricle to 4th ventricle
B. Drain into cerebral lymphatics
C. Rate not exceed 100ml/24 hours
D. Formed by choroid plexus only

123. Athlete different from normal person


A. High resting cardiac output
B. Increase oxygen consumption
C. Decrease muscle mass

153

D. Increase resting heart rate

124. Carcinogen of bronchogenic carcinoma


A. Asbestos
B. Tobacco
C. Silica
D. Nickel

125. Collagen for strength in wound healing


A. Type 1
B. Type 2
C. Type 3
D. Type 4

126. 3rd ventricle


A. Communicate with 4th ventricle through interventricular foramina
B. Inferiorly transverse fissure
C. Posterior to lamina terminalis
D. Recessed into mammillary body

127. Right Atrium


A. Fossa ovalis in AV wall
B. Related to Diaphragm at t10
C. Coronary sinus at posterior part of AV groove

128. Which contains more content of carbohydrate


A. Rice

154

B. Maize
C. Sorghum
D. Cereals

129. Parotid acini, under microscope dark reddish colour and cytoplasm basophilia is due to
A. Lysosomes
B. SER
C. Mitochondria
D. RER
E. Golgi bodies

130. Important factor in pathogenesis of thrombosis


A. Endothelial injury
B. Prolong bed rest
C. Atrial fibrillation
D. Antithrombin 3 deficiency
E. OCPS

131. Both alleles are fully expressed in heterozygote called


A. Codominance
B. Transduction
C. Polymorphism
D. Reduce pentance
E. Variable expressivity

132. Fracture of long bone will stop longitudinal growth


A. Epiphyseal plate

155

B. Epiphyseal line
C. Anatomical neck
D. Metaphysis
E. Diaphysis

133. Passage of drugs across biological membrane takes place by simple diffusion
A. Un saturable
B. Needs energy
C. Needs carrier molecule
D. Can occurs against concentration gradient
E. Can occur for water-soluble

134. Protein involved in power stroke of muscle


A. Myosin
B. Actin
C. Myoglobin
D. Tropomyocin
E. Calmodulin

135. Vasopressin and ADH originated from


A. Hypothalamus
B. Neurohypophysis
C. Adenohypophysis
D. Glia of ANS
E. Adrenal medulla

136. Measurement most useful in monitoring of progression of disease

156

A. Precision
B. Sensitivity
C. Specifity
D. Coefficient of variance
E. Predictive validity

137. A 40-year female, non-diabetic female FBS 122mmol,after 1hr 198mmol,afterr 2hr 194mmol diagnosis
A. Impaired glucose tolerance
B. Diabetes
C. Impaired Glucagon
D. Pre Diabetic

138. Stony dull percussion on right side of chest


A. Pleural effusion
B. Thickening of visceral pleura
C. Thickening of parietal pleura
D. Air in alveoli

140. Definitive evidence of malignancy


A. Local Invasion
B. Metastasis
C. Increase mitotic activity
D. Tripolar Necrosis
E. Pleomorphism

141. Lordosis is characterized by increase curve of vertebral column


A. Concave anteriorly

157

B. Concave posteriorly
C. Convex anteriorly
D. Convex to sides
E. Concave laterally

142. Man develop hypoglycemia which hormone secretion will correct it


A. Thyroxine
B. Glucagon
C. Cortisol
D. Epinephrine
E. Growth hormone

143. A baby born with erythroblastosis fetalis with blood group A+ he receive blood group
A. A+
B. AB+
C.O+
D. A-

144. Pulse pressure


A. Depends upon right ventricle stroke volume
B. Low in high cardiac output state
C. Increase in peripheral arteries than systemic
D. Depends on compliance of venous system

145. Thyroid moves on swallowing because of


A. Pretracheal fascia
B. Prevertebral fascia

158

C. Investing layer of deep cervical fascia


D. Carotid fascia
E. Superficial fascia

146. Toxins causes cell death through enzymatic level, Cyanide interfere with
A. Adenyl cyclase
B. Cytochrome oxidase
C. Golgi
D. Citric acid cycle

147. A 45 year old male presented in ER with fracture of pelvic bone in RTA with shock due to
A. Excessive blood loss
B. Bladder injury
C. Malunion
D. DVT
E. Bowel involvement

148. Stroke volume


A. Determined by heart rate
B. Unaffected by preload
C. Is Independent of myocardial contraction
D. Related normally to afterload
E. Is determined by heart rate

149. One of following lesion develop into malignancy


A. Trachoma
B. Bowen disease

159

C. Lymphocytoma
D. Inter epithelial nevus

150. In physiological limits right arterial pressure increase e


A. Increase cardiac output
B. Decrease systemic circulation
C. Decrease heart rate
D. Decrease renal blood flow
E. Increase intrathoracic pressure

151. e-max achieved by drug is measure of


A. Efficacy
B. Safety
C. Potency
D. Bioavailability
E. Intrinsic activity

152. Immunoglobin synthesis occur in


A. Plasma cells
B. Platelets
C. Fibroblasts
D. Red cells

153. In Wernicke lesion


A. Fully comprehensive
B. Short term memory
C. Clear words understandable

160

D. Rapid non sense way

154. I.V penicillin has given to 27-year-old female after few minutes she developed tachycardia
Shortness of breath, rash on body chemical mediators responsible for it
A. Complement c5a
B. Bradykinin
C. Histamine
D. Thromboxane
E. Interleukin

155. VwF cause


A. Platelets adhesion
B. Platelets aggregation
156. Pivot joint
A. Proximal tibofibular joint
B. Atlanoaxial joint

157. Secondary center ossification


A. Epiphysis
B. Metaphysis
C. Epiphyseal plate
D. Diaphysis

158.DM foot... Angiopathy and neuropathy

159. In emergency
A. Only patient allow in EM room and family stay out

161

B. Give them psychological support

160.Parietal cells secrete from.. Gastric fundus

161. Highest Cholesterol lipoprotein.. LDL

162. Normal distribution curve.. also called Gaussian curve

Remember me in your prayers ( BINT.E.ADAM)

=-=-=-=-=-=-=-=-=-=-=-=-=-=-=-=-=-=-=-=-=-=-=-=-=-=-=-=-=-=-=-=-=-=-=-=-=-=-=-=-=-=The End- Compiled by : Amlodipine Besylate


=-=-=-=-=-=-=-=-=-=-=-=-=-=-=-=-=-=-=-=-=-=-=-=-=-=-=-=-=-=-=-=-=-=-=-=-=-=-=-=-=-=-

Anda mungkin juga menyukai